curso de post-grado para profesores especialidad en

Transcripción

curso de post-grado para profesores especialidad en
curso de post-grado para profesores
especialidad en matemática
GEOMETRÍA EUCLIDEANA
Equipo de Diseño:
Nahomy Jhopselyn Hernández Cruz
Gabriel Alexander Chicas Reyes
Eduardo Arnoldo Aguilar Cañas
Héctor Enmanuel Alberti Arroyo
Ernesto Américo Hidalgo Castellanos
Juan Agustı́n Cuadra
Claudia Patricia Corcio López de Beltrán
Carlos Mauricio Canjura Linares
Oscar Armando Hernández Morales
Aarón Ernesto Ramı́rez Flores
5 de abril de 2010
Índice
1. Ángulos entre paralelas.
2
2. Triángulos: Teoremas Fundamentales.
7
3. Congruencia de Triángulos.
16
4. Cuadriláteros: Clasificación y Propiedades.
24
5. Ángulos en la Circunferencia.
30
6. Teorema de Thales y su recı́proco. Semejanza de Triángulos.
46
7. Puntos y Rectas Notables del Triángulo.
58
8. Solución a Problemas Selectos.
72
1
1.
Ángulos entre paralelas.
ÁNGULOS
Definimos como ángulo a la figura geométrica formada por dos rayos (o semirrectas) distintas
que tienen el mismo origen. Ese origen se llama vértice del ángulo. Al ángulo de vértice O y
rayos OA y OB se le denota ∠AOB.
Dos ángulos ∠AOB y ∠BOC son adyacentes si y sólo si tienen un lado común OB y los lados
no comunes OA y OC están en semiplanos distintos, determinados por el lado común.
Bisectriz de un ángulo es la semirrecta que lo “divide” en dos ángulos adyacentes iguales.
Dos ángulos son:
Congruentes o Iguales: si tienen igual medida.
Suplementarios: si su suma es 180°.
Complementarios: si su suma es 90°.
Por otra parte, dos rectas en el plano pueden ser secantes o paralelas,1 dependiendo si se cortan
o no; además, si las rectas son secantes, el punto de corte es único, y definen cuatro ángulos,
que se agrupan por parejas en ángulos opuestos por el vértice (las parejas de ángulos tales que
uno está formado por la prolongación de los lados del otro).
Los ángulos opuestos por el vértice son iguales (Justifique), por lo que dos rectas secantes forman cuatro ángulos que definen dos parejas de ángulos iguales, y si tomamos un miembro de
cada pareja, se tienen dos ángulos suplementarios. En particular, si las rectas son secantes y
forman cuatro ángulos iguales, serán llamadas rectas perpendiculares,2 y los ángulos ası́ generados son llamados ángulos rectos. Y como es muy conocido, un ángulo agudo es aquel cuya
medida es menor a la de un ángulo recto, y un ángulo obtuso es aquel cuya medida es mayor
que un ángulo recto; en particular, un ángulo obtuso será llamado ángulo llano si su medida es
el doble que la de un ángulo recto.
ÁNGULOS ENTRE PARALELAS
Al intersecar un par de rectas paralelas por una recta llamada transversal o secante, se forman
los siguientes tipos de ángulo:
Ángulos Correspondientes: Son dos ángulos no adyacentes situados en el mismo lado de
la secante, uno en el interior y otro en el exterior de las paralelas.
Ángulos Alternos Internos: Son dos ángulos no adyacentes situados en el interior de las
paralelas, y en distintos lado de la secante.
1
2
Si la recta AB es paralela a la recta CD, se denota AB k CD.
Si la recta AB es perpendicular a la recta CD, se denota AB ⊥ CD.
2
Ángulos Alternos Externos: Son dos ángulos no adyacentes situados en el exterior de las
paralelas, y en distintos lado de la secante.
Ángulos Conjugados: Son los ángulos no adyacentes situados uno en el interior y el otro
en el exterior de las rectas paralelas y del mismo lado de la secante.
Las propiedades fundamentales de los ángulos entre paralelas son:
1. Los ángulos correspondientes son iguales entre sı́.
2. Los ángulos alternos internos son iguales entre sı́.
3. Los ángulos alternos externos son iguales entre sı́.
4. Los ángulos conjugados son suplementarios.
Figura 1: Ángulos entre las rectas paralelas L1 y L2 .
Ejercicios
1. Tres ángulos adyacentes forman un semiplano y tienen sus medidas proporcionales a los
números 5, 7 y 8. Hallar la medida del menor ángulo.
2. Demostrar que las bisectrices de dos ángulos suplementarios son perpendiculares.
3. En la figura adjunta, L1 k L2 y
L3 k L4 . Calcular x.
3
4. Con ayuda de la figura 2, demuestre que: Si L1 k L2 entonces γ = α + β.
Figura 2
5. En la figura 3, AB k F G. Hallar el ángulo x si el ∠AM F = 90° y el ∠M AB = 110°.
Figura 3
6. Calcular el ∠OP Q, si OP es bisectriz del ángulo O, L1 k L2 y P Q ⊥ L1 . Ver figura 4.
Figura 4
4
7. En la figura 5, L1 k L2 y L3 k L4 , calcular α.
Figura 5
8. En la figura 6, calcular x, si L1 k L2 .
Figura 6
9. Calcular la medida θ del gráfico anexo, si las
rectas L1 y L2 son paralelas.
5
10. En la figura 7, L1 k L2 y L3 k L4 . Hallar el valor del ángulo θ.
Figura 7
11. Sea ∠AOB = 24°, en la región exterior a dicho ángulo se traza el rayo OC. Hallar la
medida del ángulo formado por las bisectrices de los ángulos AOC y BOC.
12. Del gráfico 8, calcular y, cuando x tome su máximo valor entero.
Figura 8
6
2.
Triángulos: Teoremas Fundamentales.
TEOREMAS FUNDAMENTALES EN TODO TRIÁNGULO.
Diremos que tres puntos que pertenecen a una misma recta son puntos colineales; de manera
análoga, si tres rectas pasan por un mismo punto, serán llamadas rectas concurrentes. Si tomamos “al azar” tres puntos en el plano, en muy raras ocasiones estos puntos estarán alineados,3
y diremos entonces que son los vértices de un triángulo; análogamente sucede con las rectas,
tres rectas por lo general no concurren, y la figura geométrica que éstas definen es también un
triángulo.4 Una definición completa para nuestros intereses es la siguiente:
Definición de Triángulo. Si A, B y C son tres puntos cualesquiera no colineales (Ver figura
9), entonces la reunión se los segmentos AB, BC y AC se llama triángulo ABC y se denota
por 4ABC. Los puntos A, B y C se llaman vértices y los segmentos AB, BC y AC se llaman
lados. Simbólicamente: 4ABC = AB ∪ BC ∪ AC. Todo triángulo ABC determina tres ángulos
internos o interiores: ∠ABC, ∠ACB y ∠BAC, y se llamará ángulo externo o exterior, al
ángulo determinado por un lado y la prolongación del lado adyacente, en la figura 9, α, β y θ
son ángulos exteriores.
Figura 9: Elementos del Triángulo
Dado el 4ABC, se tiene que AB + BC + CA = p = 2s, donde p es llamado el perı́metro y s
el semiperı́metro del triángulo. Para abreviar, suele asociarse a cada vértice un lado opuesto,
y viceversa, por ejemplo, el lado opuesto de A es BC, y es frecuente que se denote por a;
análogamente b = CA, c = AB.
Teorema 1: En todo triángulo, la medida de un ángulo exterior es igual a la suma de las
medidas de dos ángulos interiores del triángulo no adyacentes a él.
La demostración de este teorema se basa en las relaciones de ángulos entre paralelas; se deja
al lector que haga la demostración (Sugerencia: por un vértice, trace una recta paralela al lado
3
En teorı́a de probabilidades, ¡la probabilidad que esto ocurra es cero!
El término más riguroso para esta figura es trilátero. En este caso, habrı́a que hacer una consideración: si
hay un par de rectas paralelas, el trilátero definido ya no es “normal” según nuestro sentido común, sin embargo,
¡sigue siendo un trilátero!
4
7
opuesto)
Corolario: En todo triángulo, la suma de las medidas de sus tres ángulos internos es igual a
180°.
Teorema 2: Desigualdad Triangular. En todo triángulo, la longitud de uno de sus lados
está comprendido entre la suma y la diferencia de los otros dos.
Sin ser muy rigurosos, suponga que dado el segmento AB se traza con centro en A una circunferencia de radio r1 , y con centro en B una circunferencia de radio r2 ; si AB < r1 + r2 ,
las circunferencias se cortarán en dos puntos, y cualquiera de ellos puede ser el vértice C,
ası́ AB < BC + CA; en cambio, si AB = r1 + r2 o peor aún, si AB > r1 + r2 , la construcción
del 4ABC no es posible.
La Desigualdad Triangular es un resultado fundamental, a partir de ésta y de su modelo de
demostración se generan los Criterios de Congruencia de Triángulos; a groso modo, si dadas
ciertas condiciones, la construcción de una figura geométrica (un triángulo en particular) queda
determinada de manera única, entonces dos figuras que reunen las mismas condiciones serán
llamadas figuras congruentes.
Ası́, si se tienen tres segmentos (cuyas longitudes cumplen la desigualdad triangular), dejando
uno fijo y construyendo las circunferencias con centros en los extremos de este segmento y
radios las longitudes de los otros segmentos, por construcción, sólo será posible obtener dos
triángulos (uno con cada punto de intersección de las circunferencias), que son básicamente el
mismo pero la orientación de los ángulos es contraria; ası́, si se sabe que dos triángulos cumplen
tener lados respectivamente iguales, por construcción, deben de ser iguales. Este es el conocido
criterio LLL de congruencia de triángulos; más adelante se detallarán el resto de criterios, pero
a partir de este probaremos el siguiente resultado:
Teorema 3: En todo triángulo, se cumple que a lados iguales se oponen ángulos iguales, y
viceversa.
Suponga que 4ABC es tal que AB = AC, entonces, por criterio LLL, 4ABC es congruente
al 4ACB (en ese orden, porque AB = AC, BC = CB y CA = BA), entonces, los ángulos
que se oponen a los ángulos iguales son iguales. Para el recı́proco necesitamos otro criterio de
congruencia, por lo que la demostración se dejará incompleta; retome esto en la sección de
congruencia de triángulos.
Teorema 4: En todo triángulo se cumple que a mayor lado se opone mayor ángulo y viceversa.
Este teorema se deja como ejercicio para el lector (Sugerencia: utilice el teorema anterior, tome
el lado mayor y defina un punto adecuado que genere un triángulo con dos lados iguales.)
CLASIFICACIÓN DE TRIÁNGULOS.
1. Con relación a sus lados:
8
a) Escaleno: si sus tres lados no son congruentes.
b) Isósceles: si por lo menos dos de sus lados son congruentes.
c) Equilátero: si sus tres lados son congruentes (note un triángulo equilátero es también
isósceles, y que los tres ángulos internos son iguales entre sı́ e iguales a 60)
2. Con relación a sus ángulos internos:
a) Acutángulo: si su ángulo mayor es agudo (note que entonces los tres ángulos son
agudos)
b) Rectángulo: si su ángulo mayor es ángulo recto (note que el ángulo en cuestión es
único y que los otros dos ángulos son agudos; ası́, en un triángulo rectángulo, la
hipotenusa es mayor a los catetos)
c) Obtusángulo, si el ángulo mayor es ángulo obtuso (note que el ángulo en cuestión
es único y que los otros son agudos; ası́, en un triángulo obtusángulo, el lado que se
opone al ángulo obtuso es el lado mayor)
LÍNEAS NOTABLES EN UN TRIÁNGULO.
1. Altura: Se llama altura de un triángulo al segmento que parte de uno de sus vértices y
llega en forma perpendicular al lado opuesto o a su prolongación.
2. Mediana: Se llama Mediana al segmento que une un vértice con el punto medio del lado
opuesto.
3. Mediatriz: Se denomina mediatriz de un lado de un triángulo es la recta perpendicular a
dicho lado en su punto medio.
4. Una Bisectriz: La bisectriz es la recta que “divide” en dos ángulos iguales a un ángulo
dado; en particular, es bisectriz interna si es la bisectriz de un ángulo interno de un
triángulo, y bisectriz externa si es la bisectriz de un ángulo externo de un triángulo.
DISTANCIA DE UN PUNTO A UNA RECTA.
En la figura 10, sea P un punto exterior a una recta L, la longitud de la perpendicular P M a
la recta L es la distancia del punto P a dicha recta. Esta perpendicular tiene la propiedad de
ser única y su longitud es la distancia mı́nima del punto a la recta (Pruébelo utilizano el hecho
que la hipotenusa es mayor que los catetos).
Los segmentos P A y P B no son perpendiculares a L y se llaman oblicuas.
TEOREMA DE PITÁGORAS.
Abordamos el estudio de las Relaciones Métricas, del cual solo realizaremos el análisis del famoso Teorema de Pitágoras, cuyo enunciado es el siguiente:
Teorema: Pitágoras. En un triángulo rectángulo, el cuadrado de la hipotenusa es igual a la
suma de los cuadrados de los catetos.
9
Figura 10
Una demostración de este teorema es debida a Thabit ibn Qurra (836-901), la cual consiste en
diseccionar la figura que se forma al construir dos cuadrados de lados respectivamente iguales
a los catetos de un triángulo rectángulo, como se muestra en el gráfico 11.
Figura 11
Recı́proco del teorema de Pitágoras: Si en un triángulo el cuadrado de un lado es igual a
la suma de los cuadrados de los otros dos lados, el triángulo es rectángulo.5
Ejercicios.
1. En la figura adjunta ambos triángulos son
equiláteros. Encuentre el valor de ϕ.
5
Ver demostración en la sección de congruencia de triángulos.
10
2. En la figura 12, calcular el ∠x si el ∠AOB = 100° y L1 k L2 .
Figura 12
3. (*) En la figura 13, ABDE es un cuadrado y BCD es un triángulo isósceles con BD =
DC. Si ∠ABC = 160°, determinar la medida de ∠AEC.
Figura 13
4. (*) (XV Competencia de Clubes Cabri Primera Ronda) En la figura
adjunta, ABCD es un rectángulo tal que AB = 2BC. M es el punto
medio de AB y los triángulos AM E y M BF son equiláteros. Si P
es la intersección de las rectas DE y CF , encuentre los ángulos del
4CDP .
5. Si AB y F G son rectas paralelas, el ∠ABC = ∠CDE = θ, el ∠DEF =
150°. Calcule θ. Figura 14
θ
2
y el ∠GF H =
6. Probar que una bisectriz exterior de un triángulo es paralela al lado opuesto si y sólo si
el triángulo es isósceles.
11
Figura 14
7. (*) Hallar la suma de los ángulos α + + θ + φ en la figura 15.
Figura 15
8. Determine el valor de la suma ∠A + ∠B + ∠I + ∠H + ∠F + ∠G. Figura 16.
Figura 16
12
9. En el 4ABC el ∠BAC = 36° y AC = AB. Probar que la bisectriz interior BD, D en
AC, es congruente con el lado BC.
10. Sea ABC un triángulo rectángulo en B con AB = BC, se construye exteriormente el
triángulo equilátero BCD. Encuentre el ángulo ∠DAB.
11. En el 4ABC, AB = AC y D un punto sobre la recta AC, tal que BC = BD = DA.
Determine la medida del ángulo ∠ABD, si:
a) D está entre A y C.
b) A está entre D y C.
12. En un 4ABC, D es un punto sobre el lado AC tal que AB = AD. Si ∠ABC − ∠ACB =
90°, hallar el ∠CBD.
13. En la figura 17, el ∠ABC = ∠ACE, DC = EC, ¿Qué lı́nea notable es AD del 4BCA?
Figura 17
14. Se tiene un triángulo isósceles ABC, AB = BC en el cual se traza al altura AF tal que
BF = 6 y F C = 2. Hallar AC.
15. ¿Cuál es el valor de b − a en la figura 18?
Figura 18
13
16. La hipotenusa BC de un triángulo rectángulo ABC se divide en 4 segmentos congruentes
por los puntos G, E y H. Si BC = 20, encuentra la suma de los cuadrados de las longitudes
de los segmentos AG, AE y AH. Figura 19.
Figura 19
17. (*) Dado un cuadrado ABCD, se construyen los triángulos equiláteros ABP (exteriormente) y ADQ (interiormente). Probar que C, P y Q están alineados.
18. (*) Sea ABC un triángulo rectángulo con ∠CAB = 90°. D es un punto sobre la prolongación de BC tal que BD = BA. E es un punto en el mismo semiplano que A respecto
de BC, tal que CE ⊥ BC y además CE = CA. Mostrar que A, D y E están alineados.
19. El cuadrilátero ABCD mostrado en la figura 20 cumple que AB k CD y BC k DA.6
Sobre las prolongaciones de AB y AD se construyen puntos E y F tales que BC = BE
y DC = DF . Demuestre que C, E y F están alinedos.
Figura 20
20. (*) En la figura adjunta, AB = BC = CD =
DE = EF = F G = GA. Calcule la medida del
∠DAE.
6
El cuadrilátero ABCD es un paralelogramo.
14
21. (*) (XXVIII Olimpiada Brasileña de Matemática) En la figura 21, AB = AC, AM = AN
y ∠CAM = 30°, encuentre el valor del ∠BM N .
Figura 21
22. Los lados de un triángulo isósceles son 12 y 5 metros, ¿cuál es su perı́metro?
23. Muestre que los lados de un triángulo cumplen que |a − b| < c y que c <
a+b+c
.
2
24. Muestre que es posible construir un triángulo con segmentos de longitudes a, b, c si y sólo
existen números positivos x, y, z tales que: a = x + y, b = y + z, c = z + x.
Problemas de Refuerzo.
25. (*) (Etapa semifinal Estatal de XXII Olimpiada Mexicana de Matemáticas) En la figura
22 se muestra un hexágono regular ABCDEF de lado 1. Los arcos del cı́rculo que están
dibujados tienen centro en cada vértice del hexágono y radio igual a la distancia al vértice
opuesto. P , Q, R, S, T y U son los puntos de corte de estos arcos. ¿Cuánto mide cada
lado del hexágono P QRST U ?
Figura 22
15
3.
Congruencia de Triángulos.
CRITERIOS DE CONGRUENCIA.
Definición de Congruencia de triángulos. El 4ABC es congruente al 4A0 B 0 C 0 si: AB =
A0 B 0 , AC = A0 C 0 , BC = B 0 C 0 , ∠ABC = ∠A0 B 0 C 0 , ∠ACB = ∠A0 C 0 B 0 y ∠BAC = ∠B 0 A0 C 0 .
Simbólicamente: 4ABC = 4A0 B 0 C 0 . Véase figura 23.
Figura 23: Definición de Igualdad de Triángulos.
La definición anterior establece que dos triángulos son congruentes si tanto los lados como
los ángulos se presentan en pares respectivos congruentes. Esto, según la visión de Euclides,
significa que un triángulo es posible superponerlo sobre el otro (se puede desplazar, girar o
reflejar) y coincidirá de manera perfecta. Sin embargo, es importante mencionar que en muy
raras ocasiones se tendrá a disposición tanta información, de allı́ la importancia de los criterios
de congruencia, que establecen los requisitos mı́nimos para garantizar que dos triángulos son
congruentes.
El siguiente es el primero de los tres criterios de congruencia de triángulos, y se denomina
criterio de LADO-ÁNGULO-LADO, en sı́mbolos: L-A-L.
Criterio L-A-L. Si los triángulos ABC y A0 B 0 C 0 presentan las congruencias: AB = A0 B 0 ,
AC = A0 C 0 y ∠BAC = ∠B 0 A0 C 0 , entonces 4ABC = 4A0 B 0 C 0 .
Figura 24: Criterio LAL
Según el criterio L-A-L, dos triángulos son congruentes si en uno de ellos existen dos lados y el
ángulo (comprendido entre dichos lados), respectivamente congruentes a dos lados y el ángulo
(comprendido entre dichos lados), en el otro triángulo.
Criterio A-L-A. Sean ABC y A0 B 0 C 0 dos triángulos tales que: AC = A0 C 0 , ∠BCA = ∠B 0 C 0 A0
y ∠BAC = ∠B 0 A0 C 0 , entonces 4ABC = 4A0 B 0 C 0 .
16
Figura 25: Criterio ALA.
Criterio L-L-L. Si un triángulo tiene sus tres lados respectivamente congruentes a los tres
lados de otro triángulo, entonces estos dos triángulos son congruentes.
Figura 26: Criterio LLL.
Ahora demostraremos el Recı́proco del Teorema de Pitágoras.
Demostración: Sea ABC un triángulo talque BC 2 = AB 2 + AC 2 , por construcción sea el
4A0 B 0 C 0 rectángulo en A0 tal que A0 B 0 = AB y A0 C 0 = AC, entonces por el teorema de
Pitágoras B 0 C 02 = A0 B 02 + A0 C 02 , ası́ que B 0 C 02 = BC 2 , de donde B 0 C 0 = BC y por el criterio
LLL, se deduce que el 4A0 B 0 C 0 = 4ABC, por lo tanto el ∠BAC = ∠B 0 A0 C 0 = 90°.
TEOREMA DE LA BASE MEDIA
En todo triángulo, el segmento que une los puntos medios de dos lados es paralelo al tercer
lado e igual a su mitad.
En la figura 27, M N es el segmento que une los puntos medios de los lados AB y BC del 4ABC,
AC
a este segmento se le llama BASE MEDIA DEL TRIÁNGULO. Se verifica que M N =
y
2
que M N k AC.
Demostración:
1. Prolongar el segmento M N hasta el punto P tal que M N = N P .
2. Los triángulos M N B y P N C son congruentes, ya que BN = N C, M N = N P y el
∠P N C = ∠BN M , por consiguiente, el ∠N CP = ∠M BN , por lo tanto, CP k M B (Por
ángulos alternos internos iguales). Además, P C = M B = M A; con lo cual se tiene que:
M A = P C.
17
Figura 27: Teorema de La Base Media.
Figura 28: Menor Media en un Triángulo Rectángulo.
3. Uniendo el punto A con el punto P se forman los triángulos congruentes AM P y ACP
(por L A L) ya que M A = P C, AP = AP , ∠M AP = ∠AP C (por ángulos alternos
internos entre las paralelas M A y P C). Luego, M P = AC, entonces N P = 12 M P = 12 AC.
Además, ∠P AC = ∠M P A, de donde M P k AC o que M N k AC.
Corolario: Menor mediana de un triángulo rectángulo. En todo triángulo rectángulo,
la mediana relativa a la hipotenusa es la mitad de la longitud de la hipotenusa y es la menor
de las tres medianas del triángulo.
Demostración: En la figura 28, BM es la mediana relativa a la hipotenusa AC del 4ABC,
; (con lo cual se tendrá que BM = AM = M C). Si por M se traza
probaremos que BM = AC
2
una paralela al lado AB, que corte al lado BC en N , entonces N es el punto medio de BC y
el ∠M N C = 90°, los triángulos BN M y CN M son congruentes por el criterio L-A-L, luego
M B = M C = AM .
Probar que BM es la menor mediana (Ejercicio).
18
Ejercicios.
1. (*) En la figura adjunta, ABC es un triángulo equilátero
y CDEF es un cuadrado. Se construye un punto G tal
que CF = CG y además ∠CF G = 15°. Probar que
∠AGC = ∠BDC.
2. Dado un triángulo equilátero ABC, se construye un triángulo equilátero DEF cuyos
vértices están sobre los lados del 4ABC, tal como muestra la figura 29. Demuestre que
los triángulos ADF , BED, CF E son todos congruentes entre si.
Figura 29
3. ABCD es un cuadrado, E, F , G y H son puntos sobre los lados AB, BC, CD, DA, respectivamente, tal que EF GH también es cuadrado. Demuestre que los triángulos AEH,
BF E, CGF , DHG son todos congruentes entre si. Figura 30.
Figura 30
19
4. ABCDE y F GHIJ son pentágonos regulares (Vease figura 31). Demuestre que los
triángulos AF J, BGF , CHG, DIH, EJI son todos congruentes entre si.
Figura 31
5. Si AB k CD y AB = CD entonces, AD = BC y AD k BC 7 .
6. Demuestre que dos triángulos desplazados
son congruentes. Sugencia: Utilice el problema anterior.
7. Demuestre que dos triángulos rotados son congruentes.
7
El cuadrilátero ABCD se denomina paralelogramo.
20
8. Demuestre que dos triángulos reflejados con respecto a un punto
8
son congruentes.
9. Demuestre que dos triángulos reflejados con respecto a
una recta son congruentes.
Importante: Las traslaciones, rotaciones y reflexiones no cambian el tamaño ni la forma
de un triángulo.
10. (*) En la figura adjunta, ABCD un cuadrado y EF ⊥ GH.
Demuestre que que EF = GH.
11. Dos cuadrados ABCD y EHGF , ambos de lado l,
están colocados en manera tal que un vértice de uno
está en el centro del otro (como en la figura anexa).
l2
Demuestre que el área del cuadrilátero EJBK es
4
y por ende no depende de la posición de J (o K).
8
La reflexión con respecto a un punto es equivalente a una rotación de 180°
21
12. En un 4ABC el ∠B = 2∠C, la mediatriz del lado AC corta en F al lado BC. Hallar
AB, si F C = 9.
13. (*) (Examen final de XVI Olimpiada mexicana de Matemática) Los ángulos de un triángulo ABC están en progresión aritmética (∠B − ∠A = ∠C − ∠B = θ), D, E, y F son los
puntos medios de los lados BC, CA y AB, respectivamente. Llamamos H al pie de la
altura trazada desde C (que cae entre B y F ) y G a la intersección entre DH y EF .
¿Cuánto vale ∠F GH?
14. En la figura 32, AC = 12 AF = 4 y ∠BAF = 30. Hallar BF si AG = GC.
Figura 32
15. En la figura 33, AG = GC, el ∠AF G = 20°. Hallar el ∠F AC, si AC = 2BF .
Figura 33
16. (*) Sea ABCD un cuadrado. Se construyen triángulos equiláteros ADP y ABQ como se
muestra en la figura 34. Sea M la intersección de CQ con AD y N la intersección de CP
con AB. Demuestre que CM N es un triángulo equilátero.
22
Figura 34
Problemas de Refuerzo.
17. En la figura 35, ABC, CDE y EF A son triángulos isósceles, con el ∠ABC = ∠CDE =
∠EF A = 120°. Probar que el 4BDF es equilátero.
Figura 35
18. (*) 4ABC es un triángulo isósceles con ∠ABC = ∠ACB = 80°. D es un punto en AC
tal que ∠ABD = 10°. Demuestre que AD = BC.
23
4.
Cuadriláteros: Clasificación y Propiedades.
CLASIFICACIÓN.
Los cuadriláteros pueden clasificarse de acuerdo a sus diagonales de la siguiente forma:
Cuadrilátero Convexo: Es un cuadrilátero con las dos diagonales en su interior.
Cuadrilátero Entrante: Es un cuadrilátero con una diagonal en el interior y otra en el exterior.
Cuadrilátero Cruzado Es un cuadrilátero con las diagonales en su exterior.9
Es muy frecuente que se considere que un cuadrilátero es convexo, a menos que se especifique lo
contrario. Esto es ası́ porque muchos resultados son más claros en un cuadrilátero convexo, sin
embargo, es importante darse cuenta que existen teoremas que no se cumplen para cualquier
tipo de cuadriláteros, por ejemplo:
Teorema: La suma de los ángulos internos de un cuadrilátero no cruzado es 360.
La demostración de este resultado se basa en la disección del cuadrilátero en dos triángulos cuyos ángulos internos conforman los ángulos internos del cuadrilátero, sin embargo, estas
condiciones no pueden lograrse en un cuadrilátero cruzado; de hecho, la suma de los ángulos
internos puede hacerse arbitrariamente pequeña cuando el cuadrilátero es cruzado.
También hay otras clasificaciones de cuadriláteros de acuerdo a sus lados y ángulos.
Cuadrilátero Equiángulo: un cuadrilátero (convexo) es equiángulo si todos sus ángulos internos son iguales; dado el teorema anterior, los ángulos son iguales a 90, por ello este cuadrilátero
es llamado rectángulo.
Cuadrilátero Equilátero: un cuadrilátero (convexo) es equilátero si todos sus lados son iguales. A este cuadirátero también se le conoce como rombo.
Cuadrado: es un cuadrilátero que es equiángulo y equilátero.
Paralelogramo: es un cuadrilátero con los lados opuestos paralelos.
Trapecio: es un cuadrilátero con un par de lados opuestos paralelos.10
9
Tanto los cuadriláteros convexos como los entrantes son cuadriláteros simples, que son los cuadriláteros
cuyos lados no se cortan salvo en los extrenos; en contraposición, los cuadriláteros cruzados no son simples.
10
Note que un paralelogramo es también un trapecio.
24
PARALELOGRAMOS
Dado el paralogramo ABCD, por propiedades de ángulos entre paralelas es posible probar el
siguiente resultado:
Teorema: Los ángulos opuestos son iguales y los ángulos consecutivos son suplementarios:
∠ABC = ∠CDA = θ y ∠BCD = ∠DAB = 180 − θ.
Por otra parte, por criterio ALA, 4ABC ≡ 4CDA; esto implica que AB = CD y BC = DA,
i.e.
Teorema: Los lados opuestos de un paralogramos son iguales.
A partir de esto, si M es la intersección de AC con BD, por criterio ALA, 4ABM ≡ 4CDM ,
por lo que AM = CM y BM = DM , i.e.
Teorema: Las diagonales de un paralelogramo se bisecan.
Además, se cumple un resultado sofisticado y muy importante:
Teorema: Ley del Paralelogramo. Si ABCD es un paralelogramo entonces el doble de la
suma de los cuadrados de los lados es igual a la suma de los cuadrados de las diagonales, es
decir
2 AB 2 + BC 2 = AC 2 + BD2
Demostración: Aplicando la Ley del Coseno a 4ABC y 4ABD se tiene
AC 2 = AB 2 + BC 2 − AB · BC cos θ
DB 2 = AB 2 + AD2 − AB · AD cos(180 − θ)
⇒ AC 2 + DB 2 = 2 AB 2 + BC 2 − AB · BC (cos θ + cos(180 − θ))
y dado que cos θ = − cos(180 − θ) el resultado se sigue inmediatamente.
RECTÁNGULOS
En primer lugar, es importante notar que todo rectángulo es paralelogramo (por ángulos entre paralelas), por lo que todos los resultados probados anteriormente son heredados a todo
rectángulo; pero los rectángulos tienen propiedades adicionales:
Observe que por criterio LAL, 4ABC ≡ 4ABD, por lo que AC = BD y entonces
Teorema: Las diagonales de un paralelogramo son iguales; además, el punto de intersección
de estas equidista de los cuatro vértices y por tanto es el centro de una circunferencia que pasa
por todos los vértices.
Por otra parte, observe que si se aplica la ley del paralelogramo a un rectángulo se obtiene el
Teorema de Pitágoras.
25
ROMBOS
Dado un rombo ABCD, por criterio LLL, 4ABC ≡ 4CDA, y por lo tanto ∠BAC = ∠DAC
y ∠BCA = ∠DAC, lo cual implica BC k AD y AB k CD, i.e., todo rombo ABCD es un
paralelogramo. Además, por las mismas congruencias se tiene
Teorema: Las diagonales de un rombo cumplen ser una mediatriz de la otra.
Teorema: Las diagonales de un rombo bisecan a los ángulos interiores del rombo; esto implica
que el punto de corte de las diagonales equidista de los cuatro lados del rombo y es el centro
de una circunferencia tangente a estos.
TRAPECIOS
Dado el trapecio ABCD (con AB k CD), se construyen los puntos medios de BC y DA, M y
N , respectivamente. Si el cuadrilátero M N AB se rota con centro en M y ángulo 180 se genera
un cuadrilátero M N 0 A0 C; observe que N D = N 0 A0 y N D k N 0 A0 , por lo que DN N 0 A0 es un
paralelogramo y
N N 0 = DA0
2M N = DC + CA0
2M N = DC + AB
AB + CD
⇒ MN =
2
El segmento M N es llamado base media del trapecio, y por lo recién demostrado se tiene
Teorema: La base media de un trapecio es igual a la semisuma de las bases.
Por otra parte, hay ciertos trapecios que reciben nombres particulares; el trapecio rectángulo es
aquel que las bases son perpendiculares a alguno de los otros lados; y por otra parte, el trapecio
isósceles es aquel que los lados (distintos de las bases) tienen igual longitud. 11
Ejercicios
1. Dado el trapecio ABCD con AB k CD, demuestre que la bisectriz interior del ∠A es
paralela a la bisectriz exterior del ∠D.
2. A un rombo ABCD se le construyen exteriormente los cuadrados ABEF y BCGH.
Demuestre que 4ABD = 4EBH.
3. (*) Sea ABCD un paralelogramo. Se construyen triángulos equiláteros exteriores 4CDP
y 4ADQ. Demuestre que el 4BP Q es equilátero.
4. Demuestre que las bisectrices interiores de un paralelogramo forman un rectángulo (¿qué sucede si el paralelogramo es además rombo?).
11
Los trapecios isósceles son muy importantes cuando se estudian los ángulos en la circunferencia; resulta que
un trapecio es isósceles si y sólo si los cuatro vértices se ubican sobre una misma circunferencia.
26
5. Demuestre que las bisectrices exteriores de un paralelogramo forman un rectángulo.
6. Sea ABCD un paralelogramo. La bisectriz interna del ∠CDA corta a BA en M , y la
bisectriz interna del ∠BAD corta a CD en N . Demuestre que ADN M es un rombo.
7. Demuestre que si por el punto de intersección de las diagonales de un rombo se trazan perpendiculares a los lados del rombo, entonces los puntos de intersección de dichas
perpendiculares con los lados del rombo forman un rectángulo.
8. Demuestre que las bisectrices de los ángulos definidos por las diagonales de un rombo,
cortan a los lados del rombo en cuatro puntos que forman un cuadrado.
9. En un 4ABC sea G la intersección de las medianas BB 0 y CC 0 . Sean B 00 , C 00 las reflexiones
de G respectivas a los puntos B 0 y C 0 .
a) Demuestre que AGCB 00 y AGBC 00 son paralelogramos.
b) A partir de lo anterior, demuestre que BCB 00 C 00 también es paralelogramo.
c) Demuestre que A0 pertenece a la recta AG, y concluya que las tres medianas de un
triángulo concurren en el punto G, llamado el centroide del 4ABC.
d) Demuestre que CG = 2GC 0 ; relaciones similares se cumplen para las otras dos medianas.
10. Teorema de Varignon: Dado un cuadrilátero ABCD (no necesariamente convexo), se
construyen los puntos medios L, M , N , O, P , Q, de los segmentos de recta AB, BC, CD,
DA, BD, AC, respectivamente. Figura 36.
a) Demuestre que LM N O, LP N Q, OP M Q, son paralelogramos.
b) Demuestre que LN , OM , P Q concurren en un punto, llamado el centroide del cuadrilátero ABCD.
c) Demuestre que el perı́metro de LM N O es igual a AC + BD; resultados similares se
cumplen para los otros paralelogramos.
Figura 36: Teorema de Varignon
27
11. Sea ABCD un paralelogramo tal que existe un punto E sobre el lado AB que cumple
∠CED = 90. Sean M y N los pies de las perpendiculares trazadas desde A y B hacia
DE y CE, respectivamente. Demuestre que AC, BD y M N concurren.
12. (*) (Héctor Alberti) Sea ABCD un cuadrado. Se construyen los triángulos equiláteros
BDA0 , ACB 0 , BDC 0 y ACD0 . Demuestre que el A0 B 0 C 0 D0 es también un cuadrado.
13. (*) (II Olimpiada Matemática del Cono Sur) En la figura 37 ABCD y AECF son paralelogramos. Demuestre que BEDF es paralelogramo.
Figura 37
Problemas de Refuerzo.
14. (*) ABCD es un cuadrilátero convexo y O es un punto en su interior. Sean P , Q, R, S,
los puntos medios de los lados AB, BC, CD, DA, respectivamente. Por P se traza una
paralela a OR, por Q se traza una paralela a OS, por R se traza una paralela a OP , y
por S se traza una paralela a OQ. Demuestre que estas cuatro rectas concurren.
15. (*) Un trapecio isósceles tiene diagonales perpendiculares y su área es 2010, determine su
altura.
16. (*) (IX Competencia de Clubes Cabri, Segunda Ronda) Sea ABCDEF un hexágono
regular cuyo centro es O. Se construyen los cuadrados F SOP y ORCQ. Demuestre que
AP QB y SEDR son rectángulos. Figura 38.
17. (*) Sobre los lados del 4ABC se trazan exteriormente los cuadrados ABP Q, CARS y
BCT U . Luego se trazan los paralelogramos AQA0 R, CSC 0 T y BU B 0 P .
a) Sean A00 , B 00 , C 00 los centros de los cuadrados BCT U , CARS, ABP Q, respectivamente. Demuestre que estos centros están sobre los lados del 4A0 B 0 C 0 .
b) Demuestre que AA00 , BB 00 , CC 00 concurren.
28
Figura 38
18. (*) Se dibujan cuadrados exteriores a los lados de un paralelogramo, demuestre que:
a) El cuadrilátero determinado por los centros de esos cuadrados es un cuadrado.
b) Las diagonales de ese cuadrado son concurrentes con las del paralelogramo.
19. (*) Dado un 4ABC, se construyen exteriormente los triángulos rectángulo isósceles
4ACP y 4BCQ, con AC y BC como hipotenusas. Si M es el punto medio de AB,
demuestre que el 4M P Q también es un triángulo rectángulo isósceles.
29
5.
Ángulos en la Circunferencia.
LA CIRCUNFERENCIA Y SUS ELEMENTOS
Una circunferencia es el lugar geométrico de puntos que equidistan de un punto dado, llamado
el centro de la circunferencia; la distancia de cada punto de la circunferencia al centro es el radio.
Por otra parte, todos los puntos que están a una distancia del centro menor o igual al radio
forman el cı́rculo; estos puntos quedan “al interior” o sobre la circunferencia.
Si A y B son dos puntos de una circunferencia, el segmento de recta AB define una cuerda; en
particular, si el centro de la circunferencia pertenece a la cuerda, ésta es llamada diámetro. Es
importante mencionar que para cada punto de la circunferencia existe exactamente un punto
diametralmente opuesto.
En la figura 39, se tiene una circunferencia de centro O y radio r = OA = OB = OA0 ; AB
y AA0 son cuerdas, pero AA0 es también diámetro, i.e, A0 es diametralmente opuesto a A y
viceversa. Observe que por la desigualdad triangular aplicada al triángulo isósceles 4AOB
Figura 39
AB < AO + BO
= r+r
= AA0
Si A es un punto fijo, esta desigualdad es válida para cualquier punto B sobre la circunferencia
(excepto cuando B = A0 lo cual implica AB = AA0 ). Esto quiere decir que el diámetro es la
mayor de todas las cuerdas.
A las porciones de circunferencia que quedan entre dos puntos ubicados en la circunferencia,
se les llama arcos de circunferencia; note que dos puntos sobre una circunferencia definen dos
arcos de circunferencia. También, si un ángulo tiene vértice sobre el centro de la circunferencia y
está formado por dos radios, será llamado ángulo central ; de nuevo, ∠AOB hace referencia a dos
ángulos, cuya suma es 360, y subtienden respectivamente a uno de los arcos AB. Finalmente,
si un ángulo tiene el vértice sobre la circunferencia y está formado por dos cuerdas, será llamado ángulo inscrito; en la figura anterior, ∠AA0 B es un ángulo inscrito que subtiende al arco AB.
30
Teorema: El ángulo central es el doble del ángulo inscrito que subtiende el mismo arco.
Demostración: Considere la figura 40, se demostrará que ∠AOB = 2∠AP B en los tres casos
mostrados. En la circunferencia de la izquierda, sea P 0 el punto diametralmente opuesto a P ;
observe que 4AP O y 4BP O son triángulos isósceles, y por el teorema del ángulo externo se
tiene
∠AOB =
=
=
=
=
∠AOP 0 + ∠BOP 0
(∠AP O + ∠OAP ) + (∠BP O + ∠OBP )
2∠AP O + 2∠BP O
2 (∠AP O + ∠BP O)
2∠AP B
Figura 40
El caso de la circunferencia del medio es más sencillo y se deja como ejercicio para el lector.
Para la circunferencia de la derecha, el trabajo es análogo y sólo cambia en un pequeño arreglo
algebraico
∠AOB =
=
=
=
=
∠BOP 0 − ∠AOP 0
(∠BP O + ∠OBP ) − (∠AP O + ∠OAP )
2∠BP O − 2∠AP O
2 (∠BP O − ∠AP O)
2∠AP B
Corolario: Todos los ángulos inscritos que subtienden el mismo arco son iguales (Ver figura
41). En particular, los ángulos internos son iguales a 90° si subtienden a una semicircunferencia.
Demostración: Todos los ángulos mostrados en la figura 41 son iguales a la mitad del ∠AOB,
y por tanto, son iguales entre sı́. En particular, si AB fuera un diámetro, ∠AOB = 180° y por
tanto ∠AP B = 90°. 12
Hay un par de ángulos más que son importantes: Si un punto P es interno a la circunferencia,
el ángulo de vértice P formado por dos cuerdas que pasan por P se llama ángulo interior. De
12
Observe que en cualquier triángulo rectángulo, el punto medio de la hipotenusa equidista de los tres vértices.
31
Figura 41
forma similar, si P es exterior y dos cuerdas de la circunferencia (al prolongarse) pasan por P ,
el ángulo con vértice P es llamado ángulo exterior.
Dejamos como ejercicio demostrar el siguiente teorema:
Teorema: Los ángulos interior y exterior mostrados en la figura 42 cumplen las fórmulas
siguientes:
∠BOD + ∠AOC
2
∠BOD − ∠AOC
∠AP C =
2
∠AQC =
Figura 42
CUADRILÁTEROS CÍCLICOS
Ahora suponga que sobre una circunferencia se ubican cuatro puntos A, B, C, D, como se
muestra en la figura 43. Al cuadrilátero ABCD se le llama cuadrilátero cı́clico o concı́clico.
Observe que
α β
∠ABC + ∠CDA = + = 180◦ .
2
2
32
Figura 43
Y análogamente ∠DAB +∠BCD = 180°. Esto significa que si ABCD es un cuadrilátero cı́clico
y convexo, entonces los ángulos opuestos son suplementarios. También, es posible demostrar por
contradicción el recı́proco de este resultado: si suponemos que ABCD es tal que ∠B+∠D = 180
pero no es cı́clico, se define el punto D0 como la otra intersección de AD con el circuncı́rculo
del 4ABC, y como ABCD0 es cı́clico (por construcción) entonces ∠B + ∠D0 = 180, luego,
∠D = ∠D0 , lo cual implica la contradicción CD k CD0 (rectas paralelas que se cortan en C).
Ası́, se ha demostrado el siguiente teorema:
Teorema: El cuadrilátero convexo ABCD es un cuadrilátero cı́clico si y sólo si
∠A + ∠C = 180◦ = ∠B + ∠D
También, otro criterio muy útil y cuya demostración también se basa en el corolario anterior es
Teorema: El cuadrilátero convexo ABCD es un cuadrilátero cı́clico si y sólo si se cumple
alguna de las siguientes igualdades
∠ABD
∠BCA
∠BAC
∠CAD
=
=
=
=
∠ACD
∠BDA
∠BDC
∠CBD
Es importante recalcar que NO todo cuadrilátero puede ser inscrito en una circunferencia; por
ejemplo, un paralelogramo no será cı́clico a menos que sea rectángulo.
RECTAS Y CIRCUNFERENCIAS TANGENTES A UNA CIRCUNFERENCIA
Dada una circunferencia, una recta puede ser tangente o secante a la circunferencia, dependiendo si la corta en uno o dos puntos, respectivamente; en cualquier otro caso, se dice que la
33
recta no corta a la circunferencia.13
Sea l una recta secante a la circunferencia que corta a la circunferencia en A y B (A 6= B);
como el 4AOB es isósceles, ∠OAB < 90. Recı́procamente, si por A se traza una recta l tal
que uno de los ángulos que forma con OA es menor que 90, se puede construir un punto B
sobre l tal que ∠OAB = ∠ABO < 90 y A 6= B (basta proyectar O sobre l y luego reflejar A
con respecto a este punto, el resultante es el punto B); entonces el 4AOB es isósceles, por lo
que OA = r = OB, i.e. B pertenece a la circunferencia y por tanto l corta a la circunferencia
en dos puntos distintos. Ası́
Teorema: Una recta l corta a una circunferencia de centro O en dos puntos distintos A y B si
y sólo si un ángulo entre l y OA es agudo.
Corolario: Si l es una recta tangente en A a una circunferencia de centro O, ninguno de los
ángulos entre l y OA puede ser agudo, y por tanto l ⊥ OA.
A partir de este resultado se prueban otros resultados muy conocidos y útiles, que dejamos de
ejercicios para el lector.
Teorema: Dado un punto P externo a una circunferencia de centro O, si P A y P B son segmentos tangentes a la circunferencia en A y B, respectivamente, entonces el cuadrilátero P AOB es
cı́clico y bisósceles.
Corolario: Dado un punto P externo a una circunferencia de centro O, la circunferencia de
diámetro P O corta a la circunferencia dada en dos puntos A y B tales que P A y P B son rectas
tangentes.
Definición: El ángulo semi-inscrito en una circunferencia es aquel que se forma con una cuerda
y la recta tangente en alguno de los extremos de la cuerda.
Teorema: La media del ángulo semi-inscrito definido por la cuerda AB es igual a la medida
de un ángulo inscrito que subtiende al arco AB.
Demostración: Considere la figura 44. Como AP BO es cı́clico, entonces ∠P AB = ∠P OB;
además, como P O es la mediatiz de AB, ∠P OB = ∠P OA, por lo que
∠P AB =
∠AOB
= ∠AQB
2
Por otra parte, dada una circunferencia, otra circunferencia puede ser secante o tangente a la
primera, dependiendo si la corta en uno o dos puntos, respectivamente; en cualquier otro caso
se dice que las circunferencias no se cortan.14
13
Cuando la recta es tangente a la circunferencia puede considerarse como un caso muy peculiar en el cual
los “dos” puntos de corte coinciden.
14
También acá puede considerarse a las circunferencias tangentes como un caso especial de circunferencias
secantes en el cual los puntos de corte coinciden.
34
Figura 44
Además, dos circunferencias pueden posicionarse una dentro de la otra, y claramente, la circunferencia de radio mayor es la externa mientras que otra es la interna; particularmente, si las
circunferencias tienen el mismo centro se llaman concéntricas. Finalmente, combinando estas
definciones se tienen las circunferencias tangentes exteriormente y las tangentes interiormente.
Teorema: Dadas dos circunferencias de centros O1 y O2 que se cortan en dos puntos distintos
A y B, se cumple que O1 O2 ⊥ AB.
Teorema: Si dos circunferencias de centros O1 y O2 son tangentes en A, se cumple que O1 , A
y O2 están alineados.
Teorema:
a) Dos circunferencias, una dentro de la otra, no tienen rectas tangentes en común.
b) Dos circunferencias tangentes interiormente tienen una recta tangente común.
c) Dos circunferencias secantes (en dos puntos distintos) tienen dos rectas tangentes en común.
d) Dos circunferencias tangentes exteriormente tienen tres rectas tangentes en común.
e) Dos circunferencias no secantes y tal que ninguna contiene a la otra, tienen cuatro rectas
tangentes en común.
Ejercicios
1. Si el ∠M P Q = 20, determine el valor del ∠QON en la figura
adjunta.
35
2. Dado un ángulo inscrito BAC, y su ángulo central BOC, se sabe que ∠BAC + ∠BOC =
180°. Calcular el ∠OBC.
3. En la figura 45, BCDO es un rombo. Determine el valor del ángulo θ y la medida de las
diagonales de BCDO si el radio de la circunferencia mide 6.
Figura 45
4. Un cuadrilátero cı́clico ABCD satisface ∠ABC = 2∠CDA = θ. Calcule θ.
5. En la figura 46, P R es una tangente común. Calcule el valor del ∠P QR.
Figura 46
6. En la figura adjunta, el ∠AF E = 100° y el ∠BCD =
150°. Calcule el ∠AGB.
7. Dado un ángulo ∠AOB, se trazan dos rectas l ⊥ OA y m ⊥ OB. Si P es el punto de
corte de l y m, demuestre que A, B, O, P se ubican sobre una misma circunferencia.
36
8. Las bisectrices BP y CQ del 4ABC se cortan en I. Demuestre que si ∠BAC = 60
entonces 4P QI es isósceles.
9. En la figura 47 se ha tomado un punto C sobre la circunferencia; AC y BC cortan a la
segunda circunferencia en D y E respectivamente. Probar que OC ⊥ DE.
Figura 47
10. (*) Dada la figura 48, demuestre que AB k A0 B 0 .
Figura 48
11. En la figura 49 CR es una recta tangente en C, demuestre que AB k CR.
Figura 49
37
12. Dos circunferencias Γ1 y Γ2 son tangentes (interior o exteriormente) en P (Ver figura 50).
Dos rectas que pasan por P cortan a Γ1 y Γ2 en A y C, y en B y D, respectivamente.
Demuestre que AB k CD.
Figura 50
13. (*) Dos circunferencias de centros O1 y O2 son tangentes (interna o externamente) en un
punto P ; por este punto se traza una recta que corta nuevamente a la circunferencias en
A y B, respectivamente. Demuestre que AO1 k BO2 .
14. Dos circunferencias son tangentes externamente en el punto A. Una tangente exterior
común toca a una circunferencia en B y a la otra en C. Demostrar que ∠BAC = 90°.
15. En la figura 51, DE es tangente en D, y C es el punto medio del arco AD. Encuentre el
valor del ángulo seminscrito ADE.
Figura 51
38
16. Determine el valor del ∠DCF , sabiendo BE es tangente en el punto D a la circunferencia
de centro O. Ver Figura 52.
Figura 52
17. Si el ∠AEB = 30, ∠ADE = 20 y ∠ACE = 35, calcule el ∠AF B. Véase figura 53.
Figura 53
18. Dada una circunferencia de diámetro BC, se toma un punto P en la prolongación de BC,
y se traza la tangente AP . Si AP = AB y O es el centro de la circunferencia, demuestre
que el 4AOC es equilátero.
19. (*) Dadas dos circunferencias una fuera de la otra, demuestre que las tangentes comunes
externas forman segmentos iguales; análogamente, las tangentes comunes internas forman
segmentos iguales.
20. (*) Teorema de Pithot. Demuestre que en todo cuadrilátero inscribible, la suma de
lados opuestos es igual.
21. (*) Teorema de Steiner. En todo cuadrilátero exinscrito a una circunferencia, la diferencia de las longitudes de lados opuestos es igual.
39
22. Demuestre que las mediatrices de un cuadrilátero son concurrentes si y sólo si es cı́clico.
23. Demuestre que el cuadrilátero convexo ABCD es inscribible si y sólo si los incı́rculos
respectivos del 4ABC y 4CDA son tangentes.
24. Demuestre que las bisectrices internas de un cuadrilátero son concurrentes si y sólo si es
inscribible.
25. Demuestre que todo rombo es inscribible.
26. En la figura 54, AB es una cuerda y por D se traza una recta tangente a la circunferencia
paralela a AB. Demuestre que CD es bisectriz del ∠ACB.
Figura 54
27. Determine las medidas de ∠ACB y ∠ACO de la figura 55.
Figura 55
28. Cuatro cilindros de diámetro 1 están pegados apretadamente por
una cuerda muy fina, como en la figura adjunta. Demostrar que la
cuerda tine longitud 4 + π. Demostrar también que el área sombreada entre los cilindros es 1 − π4 .
29. En la figura 56, ABCD es un trapecio isósceles con AB k CD y DA = BC = 2; tomando
DA y BC como diámetros, se construyen dos circunferencias tangentes. Si DC = 3AB,
calcule el área del trapecio.
40
Figura 56
30. La figura 57 está formada por un paralelogramo y dos circunferencia tangentes entre sı́ y
tangentes a tres lados del paralelogramo. Sabiendo que el radio de las mismas mide la
cuarta parte del lado menor del paralelogramo, calcule la razón entre el lado mayor del
paralelogramo y el radio de las circunferencias.
Figura 57
31. En la figura 58, ABCDEF es un hexágono regular y las circunferencias de centro en los
vértices son tangentes dos a dos. Si las circunferencias sobre los vértices B, D, F son
iguales, demuestre que las circunferencias restantes son iguales.
Figura 58
32. Alrededor de una circunferencia se construyen diez circunferencias tangentes a la original
y tangentes entre sı́ (Véase figura 59). Demuestre que la suma de las áreas de las diez
circunferencias es el doble del área de la circunferencia mayor.
41
Figura 59
33. (*) Teorema de Miquel: Dado un 4ABC, sean X, Y , Z puntos sobre AB, BC, CA,
respectivamente. Demuestre que los circuncı́rculos de 4AXZ, 4BY X, 4CZY tienen un
punto en común M .
34. (X OMCC - P2, Aarón) Sea ABCD un cuadrilátero concı́clico con diámetro AC, y sea
O el centro de su circunferencia. Se construyen los paralelogramos DAOE y BCOF .
Demuestre que si E y F están sobre la circunferencia entonces ABCD es rectángulo.
35. (*) Sea ABC un triángulo, y sean L y N las intersecciones de la bisectriz del ángulo A
con el lado BC y el circuncı́rculo de ABC respectivamente. Construimos la intersección
M del circuncı́rculo de ABL con el segmento AC. Prueba que los triángulos BM N y
BM C tienen la misma área.
36. (*) Sea AB el diámetro de una semicircunferencia. Se colocan los puntos M y K sobre
la semicircunferencia y sobre AB, respectivamente.15 Sea P el centro de la circunferencia
que pasa por A, K y M ; sea Q el centro de la circunferencia que pasa por B, K y M .
Demuestre que M P KQ es concı́clico.
37. (*) Las circunferencias Γ1 y Γ2 se cortan en los puntos A y B. Por el punto A se traza
una recta que corta nuevamente a las circunferencias Γ1 y Γ2 en los puntos C y D,
respectivamente. Por los puntos C y D se trazan tangentes a las circunferencias, las
cuales se cortan en el punto M . Demuestra que M CBD es cı́clico.
38. (*) El 4ABC cumple que ∠A = 90° y AB = AC. Se toma un punto E del segmento
AB, se construye interiormente un triángulo equilátero AEF . EF corta BC en I, y se
construye exteriormente un triángulo equilátero BIJ. Encuentre ∠EJB.
39. (*) En la figura 60, se sabe que ∠AO1 B − ∠AO2 B = 70◦ y además la tangente EB forma
el triángulo isósceles ABE, con AB = AE. Encuentre ∠EBC.
15
M y K son distintos de A y B.
42
Figura 60
40. (*) Dos circunferencias Γ1 y Γ2 se cortan en A y B. Una recta por A corta a Γ1 y Γ2 en C
y D, respectivamente, y la paralela a CD por B corta Γ1 y Γ2 en E y F , respectivamente.
Demuestre que 4CDB ≡ 4EAF .
41. (*) La Recta de Simson-Wallace. Sean X, Y y Z los pies de las alturas trazadas
desde un punto P en el circuncı́rculo del 4ABC hacia AB, BC y CA, respectivamente.
Demuestre que X, Y y Z están alineados.
42. (*) Sea P un punto exterior al cuadrado ABCD tal que ∠AP C = 90◦ , Q es la intersección
de AB y P C, y R el pie de la perpendicular por Q a CA. Demuestre que P , R y D están
alineados.
43. En la figura 61, ABCD es un trapecio rectángulo tal que la circunferencia de diámetro
AB (y centro O) es tangente a CD. Demostrar que O pertenece a la circunferencia de
diámetro CD y que esta circunferencia es tangente a BA.
Figura 61
44. El 4ABC es rectángulo en C, la circunferencia de centro O es tangente a cada uno de los
lados del 4ABC en los puntos P , Q y R (como se muestra en la figura 62), y se cumple
que AP = 20 y BP = 6. Calcule OP .
43
Figura 62
45. En la figura 63 se muestran tres semicircunferencias, una de diámetro AB (de centro O
y radio r), otra de diámetro AO y la última de diámetro OB. Determine la razón entre
el radio de la circunferencia tangente a estas tres semicircunferencias y r.
Figura 63
46. El segmento AB es diámetro de un semicı́rculo con centro en O. Un cı́rculo con centro
en P es tangente a AB en O y también al semicı́rculo. Otro cı́rculo con centro en Q es
tangente a AB, al semicı́rculo y al cı́rculo de centro en P . Si AB = 2, ¿cuál es el radio
del cı́rculo con centro en Q?
Figura 64
Problemas de Refuerzo.
47. Los vértices A y B de un triángulo equilátero 4ABC están sobre una circunferencia de
radio 1 y el vértice C está en el interior de la circunferencia. Un punto D (distinto de B)
que esta en la circunferencia es tal que AD = AB. La recta DC corta por segunda vez a
la circunferencia en E. Encuentre la longitud del segmento CE. Ver figura 65.
48. (*) (OIM 2002, P-4) En un triángulo escaleno ABC se traza la bisectriz interior BD, con
D sobre AC. Sean E y F puntos sobre la recta BD tales que (AE k CF ) ⊥ BD, y sea
M el punto sobre el lado BC tal que DM ⊥ BC. Demuestre que ∠EM D = ∠DM F .
44
Figura 65
49. (*) (OMCC 2003, P-2) Sea S una circunferencia y AB un diámetro de ella. Sea t la recta
tangente a S en B y considere dos puntos C y D en t tales que B este entre C y D. Sean
E y F las intersecciones de S con AC y AD y sean G y H las intersecciones de S con
CF y DE. Demuestre que AH = AG.
50. (*) (The 59th Romanian Mathematical Olympiad District Round) Considere un cuadrado
ABCD y un punto E sobre el lado AB. La diagonal AC corta al segmento DE en el punto
P . La perpendicular por P a DE corta al lado BC en F . Probar que EF = AE + CF .
51. (*) Teorema de Arquı́medes: En la figura 66, la región delimitada por tres semicircunferencias mutuamente tangentes, es conocida como cuchilla de zapatero o árbelos.
Demostrar que las circunferencias sombreadas son congruentes.
Figura 66: Teorema de Arquı́medes.
45
6.
Teorema de Thales y su recı́proco. Semejanza de Triángulos.
Introducción.
Definición
1. Razón: se llama razón, al cociente de dos cantidades, expresadas en la misma magnitud,
por ejemplo ab .
2. Proporción: se llama proporción a la igualdad de dos razones. Por ejemplo ab = dc , 16 a los
términos a y d se les llama extremos y los términos b y c se les llama medios, al término
d se le llama cuarta proporcional entre a, b y c en este orden.
Propiedades de las proporciones:
a
c
1. = si y sólo si a· c = b· d.
b
d
2.
a
c
b
d a
b
= si y sólo si = o = .
b
d
a
c c
d
3.
a
c
a±b
c±d
= si y sólo si
=
.
b
d
b
d
4.
a
c
a+b
c+d
= si y sólo si
=
.
b
d
a−b
c−d
Paralelismo y proporcionalidad.
Definición
1. Un punto P ∈ AB divide al segmento AB en una razón dada r, si
PA
PB
= r.
Figura 67
2. Sean AB y CD dos segmentos, y sean P ∈ AB y Q ∈ CD, decimos que P y Q dividen a
QA
A
AB y CD en segementos proporcionales si PP B
= QB
.
Figura 68
16
En algunos textos de geometrı́a se utiliza la notación de proporción ası́ a : b :: c : d que se lee “a es a b como
c es a d”.
46
Teorema de Thales. Si tres paralelas cortan a dos secantes entonces los segmentos que determinan en ellas son proporcionales. 17
Antes de demostrar el Teorema de Thales, se enunciarán dos teoremas que a pesar de su aparente sencillez es de mucha utilidad en problemas que involucran Áreas y Proporcionalidad.
Lema 1. Sea AB k CD. Demuestre que: (ABC) = (ABD).
Lema 2. Sea P un punto sobre el lado AB (o su prolongación) del 4ABC. Pruebe que:
AP
(AP C)
=
PB
(P BC)
.
A continuación se enuncian los pasos a seguir en la demostración del teorema de Thales.
Demostración. Sean AA0 , BB 0 y CC 0 rectas paralelas que cortan a dos secantes en los puntos
A, A0 , B, B 0 , C, C 0 respectivamente (ver figura 69).
Figura 69: Teorema de Thales
Pruebe que:
1.
AB
(ABB 0 )
=
BC
(BCB 0 )
A0 B 0
(A0 B 0 B)
2. 0 0 =
.
BC
(B 0 C 0 B)
3. (ABB 0 ) = (A0 B 0 B) y (BCB 0 ) = (B 0 C 0 B).
Con ayuda de las igualdades demostradas concluya que:
AB
A0 B 0
= 0 0.
BC
BC
Observación Importante: Utilice las propiedades de las proporciones para demostrar las
equivalencias siguientes (interprételas geométricamente):
A0 B 0
AC
A0 C 0
AC
A0 C 0
AB
= 0 0 ⇔
= 0 0 ⇔
= 0 0
BC
BC
AB
AB
BC
BC
17
El teorema de Thales puede enunciarse de manera general como sigue: Si tres o más paralelas cortan a dos
o más secantes entonces los segmentos que determinan en ellas son proporcionales.
47
Corolario (Teorema de Thales en el triángulo). Toda recta paralela a un lado de un triángulo
y que corte a los otros dos lados, divide a estos lados en segmentos proporcionales.
Recı́proco del Teorema de Thales. Si tres rectas cortan a dos secantes en segmentos proporcionales y dos de estas rectas son paralelas entonces las tres rectas son paralelas.
Demostración. Sean AA0 , BB 0 y CC 0 rectas que cortan a dos secantes en los puntos A, A0 , B,
AB
A0 B 0
B 0 , C, C 0 respectivamente, tales que AA0 k CC 0 y
= 0 0 . Por el punto B tracemos una
BC
BC
recta paralela a AA0 , la cual interseca a A0 C 0 en el punto D (ver figura 70). Entonces, por el
A0 D
A0 B 0
A0 D
AB
=
. De donde, 0 0 =
, ası́ por las propiedades
Teorema de Thales se tiene que:
BC
DC 0
BC
DC 0
0 0
0 0
AC
AC
, por lo que B 0 C 0 = B 0 D + DC 0 = DC 0 y por tanto B 0 D = 0,
de las proporciones 0 0 =
B0 C
DC 0
o equivalentemente B = D y por lo tanto, BB 0 k AA0 .
Figura 70: Recı́proco del Teorema de Thales
Corolario (Recı́proco del Teorema de Thales en el triángulo.) Si una recta intercepta
dos lados de un triángulo en segmentos proporcionales entonces la recta es paralela al tercer
lado del triángulo.
Triángulos semejantes. Decimos que el 4ABC es semejante al 4A0 B 0 C 0 (Ver figura 71), lo
cual denotamos ası́ ABC ∼ A0 B 0 C 0 , si:
AB
AC
BC
=
=
A0 B 0
A0 C 0
B0C 0
y
∠BAC = ∠B 0 A0 C 0 , ∠ABC = ∠A0 B 0 C 0 , ∠ACB = ∠A0 C 0 B 0 .
En los tres teoremas que se muestran a continuación (los cuales son una consecuencia directa del
Teorema de Thales) se establecen las condiciones mı́nimas para demostrar que dos triángulos
son semejantes, a los cuales denominaremos: Criterios de Semejanza de Triángulos.
Primer criterio de semejanza de triángulos: Angulo-Angulo A-A. Si dos ángulos de
un triángulo son congruentes con dos ángulos de otro triángulo, entonces los dos triángulos son
semejantes.
48
Figura 71: Definición de Semajanza de Triángulos.
Demostración. Supongamos que en el 4ABC y 4A0 B 0 C 0 se tiene que ∠ABC = ∠A0 B 0 C 0
y ∠ACB = ∠A0 C 0 B 0 , entonces ∠BAC = ∠B 0 A0 C 0 (Por la suma de ángulos internos en un
triángulo).
Sea D ∈ AB y E ∈ AC tales que AD = A0 B 0 y AE = A0 C 0 , dado que ∠DAE = ∠BAC =
∠B 0 A0 C 0 , se sigue por L-A-L que 4ADE = 4A0 B 0 C 0 , por consiguiente ∠ADE = ∠A0 B 0 C 0 =
∠ABC, de donde DE k BC (por ser iguales los ángulos correspondientes) y por el teorema de
Thales
AC
AB
=
AD
AE
y por consiguiente
AC
AB
= 0 0
(1)
0
0
AB
AC
Sea F ∈ BC tal que DF k AC, entonces F C = DE = B 0 C 0 (porque DECF es paralelogramo
y por ser 4ADE = 4A0 B 0 C 0 ) y por el teorema de Thales
BA
BC
=
DA
FC
o lo que es lo mismo
AB
BC
= 0 0
0
0
AB
BC
(2)
Luego, de (1) y (2) se tiene que:
AB
AC
BC
=
=
.
A0 B 0
A0 C 0
B0C 0
Ası́, se ha demostrado que los tres pares de ángulos son congruentes y los tres pares de lados
son proporcionales, por lo tanto, 4ABC ∼ 4A0 B 0 C 0 .
Segundo criterio de semejanza de triángulos: L-A-L. Si un ángulo de un triángulo es
congruente con otro ángulo de otro triángulo y los lados que comprenden al ángulo en el primer
triángulo son respectivamente proporcionales a los lados que comprende al ángulo en el segundo
triángulo, entonces los dos triángulos son semejantes.
AB
AC
Demostración. Suponga que el ∠BAC = ∠B 0 A0 C 0 y que 0 0 = 0 0 . Considere los punAB
AC
tos D y E, como en la demostración del teorema anterior. Entonces por el criterio L-A-L,
49
4ADE = 4A0 B 0 C 0 , de lo cual se deduce que ∠ADE = ∠A0 B 0 C 0 . Por otra parte teneAB
AC
mos que:
=
, y al aplicar el recı́proco del teorema de Thales, se puede afirmar que
AD
AE
DE k BC, de lo cual a su vez se deduce que ∠ADE = ∠ABC, por ángulos correspondientes
entre paralelas. Finalmente por transitividad se concluye que ∠ABC = ∠A0 B 0 C 0 . Por lo tanto,
4ABC ∼ 4A0 B 0 C 0 (Por el criterio A-A.)
Tercer criterio de semejanza de triángulos: L-L-L. Si los tres lados de un triángulo son
respectivamente proporcionales a los tres lados de otro triángulo, entonces los dos triángulos
son semejantes.
AC
BC
AB
Demostración. Por hipótesis se tiene que: 0 0 = 0 0 = 0 0 y como antes sean D y E
AB
AC
BC
puntos sobre AB y AC respectivamente tales que AD = A0 B 0 y AE = A0 C 0 . Entonces por
el recı́proco del teorema de Thales se tiene que DE k BC y por consiguiente el ∠ABC =
∠ADE y el ∠ACB = ∠AED, de donde 4ABC ∼ 4ADE (por el criterio A-A). Por ende
BC
BC
BC
AB
=
, luego por transitividad
= 0 0 , de donde DE = B 0 C 0 . En consecuencia
AD
DE
DE
BC
4ADE = 4A0 B 0 C 0 (por el criterio L-L-L), de lo cual se sigue que ∠A0 B 0 C 0 = ∠ADE y
∠A0 C 0 B 0 = ∠AED, y por transitividad ∠A0 B 0 C 0 = ∠ABC y ∠A0 C 0 B 0 = ∠ACB =. Por lo
tanto, 4A0 B 0 C 0 ∼ 4ABC (Por el criterio A-A.)
Ejercicios
1. Sean AB y CD las bases del trapecio ABCD, cuyas diagonales son perpendiculares. Si
se sabe que AD = 13, AE = 12 y CE = 4 encuentre las longitudes de CD y AB.
2. En la figura 72, el 4ABC es equilátero, sus lados tienen longitud 3 y P A es paralela a
BC. Si P Q = QR = RS, encontrar la longitud de CS.
Figura 72
3. Sea ABCD un trapecio de bases BC y AD, sus diagonales se cortan en E. Si BE = 3,
ED = 4 y CE = 2, determine la medida de AE.
4. Las bases de un trapecio miden 3 y 5, y si su altura mide 4. Encontrar la distancia desde
el punto de corte de las diagonales hasta la base mayor.
50
5. En la figura adjunta, el 4ABC es rectángulo en A y el 4ADB
es rectángulo en D. El punto E es el punto de intersección de
los segmentos AD y BC. Si AC = 15, AD = 16 y BD = 12,
calcule el área del 4ABE.
6. El 4ABC es rectángulo en B. Se dibuja un rectángulo BEDF con D sobre la hipotenusa,
1
BC
= 1−DE
.
E y F sobre BC y AB, respectivamente. Si AB = 1, demuestre que BE
7. Considérese los puntos A, B, C y D tales que A y B están sobre el segmento OC y
OD respectivamente, donde O es el centro de la circunferencia de radio r (Ver figura
73). Si OA·
OC = r2 = OB· OD, demuestre que el 4AOB ' 4DOC y que CD =
r2
AB.18
OA· OB
Figura 73
8. Sobre la circunferencia de centro O, se trazan los diámetros AB y CD tales que AB ⊥ CD.
Sea P un punto sobre el arco CBD y Q el punto de intersección de las cuerdas AP y
CD. Si DO = 1, demuestre que AP · AQ = 2.
9. Un segmento de recta AB es divido por los puntos interiores K y L de manera que
AL2 = AK· AB. Sea P un punto exterior al segmento AB tal que AP = AL. Pruebe que
∠KP L = ∠LP B. Figura 74.
Figura 74
18
La medida del segmento CD se denomina Distancia Inversa.
51
10. En la figura 75, AB y AC son tangentes a la circunferencia, y CE ⊥ BD, siendo BD un
diámetro. Probar que BE ·BO = AB ·CE.
Figura 75
11. Demostrar que
1
1
1
+
=
si se cumple que AX k BY k CZ. (Ver figura 76.)
AX BY
AZ
Figura 76
12. En la figura 77, el 4ABC es rectángulo. Se construyen exteriormente los cuadrados
ABEF y BCP Q. Demostrar que BM = BN .
Figura 77: .
52
13. Sean O, P y R los centros de las tres circunferencias. Si OR = r y Q es la intersección
de P O con la circunferencia de centro R, demuestre que OP · OQ = r2 . Ver figura 78.
Figura 78
14. Si en un triángulo rectángulo se traza la altura correspondiente a la hipotenusa, entonces:
a) Los dos nuevos triángulos que resultan, son semejantes entre si y semejantes al
triángulo original.
b) La altura es media proporcional
hipotenusa.
19
entre los segmentos que ella determina sobre la
c) Cada cateto es media proporcional entre la hipotenusa y la proyección del cateto
sobre la hipotenusa.
d ) Demuestre el teorema de Pitágoras.
15. Si dos triángulos tienen sus lados respectivamente paralelos o respectivamente perpendiculares, entonces los dos triángulos son semejantes.
16. Las alturas, las bisectrices y las medianas homólogas de dos triángulos semejantes están
en la misma razón que sus lados homólogos.
BC
CA
17. Sean ABC y A0 B 0 C 0 dos triángulos semejantes con AAB
0 B 0 = B 0 C 0 = C 0 A0 = k. Demuestre
que: la razón entre los perı́metros de los triángulos es k y que la razón entre sus áreas es
k2.
18. Teorema de Menelao. Dado el 4ABC, sea P un punto sobre la recta AB, Q un punto
sobre la recta BC, R un punto sobre la recta CA. Si los puntos P , Q, R están alineados
AP BQ CR
entonces
= 1.
P B QC RA
Para demostrar este teorema, sea W un punto sobre la recta P QR tal que BW k AC:
a) Demuestre que los triángulos AP R y BP W son semejantes.
Si b es una magnitud tal que ab = cb , entonces decimos que b es media proporcional entre a y c,o de manera
equivalente: b es media proporcional entre a y c si y solo si b2 = a· c.
19
53
Figura 79: Teorema de Menelao.
b) Demuestre que los triángulos CQR y BQW son semejantes.
AP BQ CR
c) De los literales a) y b) deduzca que
= 1.
P B QC RA
19. Teorema de Ceva. Dado el 4ABC, sea P un punto sobre el
recta AB, Q un punto sobre la recta BC y R un punto sobre
la recta CA. Si las rectas AQ, CP , BR concurren, entonces
AP BQ CR
= 1.
P B QC RA
Para demostrar este teorema, sean W y V los puntos de intersección de la recta que pasa
por B paralela a AC, con las rectas CP y AQ, respectivamente.
a) Demuestre que 4AP C ∼ 4BP W y que 4AQC ∼ 4V QB.
b) Demuestre que 4BW P ∼ 4RCP y que 4BV P ∼ 4RAP .
AP BQ CR
= 1.
c) Utilice los literales a) y b) para probar que
P B QC RA
20. Si dos cuerdas se interceptan en el interior de una circunferencia entonces el producto
de las medidas de los segmentos determinados por el punto de intersección en una de las
cuerdas es igual al producto de las medidas de los segmentos determinados en la otra
cuerda.
21. Si dos segmentos se interceptan en un punto que esta en el interior de los dos segmentos
y el producto de las medidas de los segmentos determinados por el punto de intersección
en el primer segmento es igual al producto de las medidas de los segmentos determinados
por el punto en el segundo segmento,entonces los extremos de los segmentos están sobre
una circunferencia.
22. Si desde un punto P exterior a una circunferencia se trazan dos semirrectas secantes
que cortan a la circunferencia en los puntos A, B y C, D respectivamente, entonces
P A· P B = P C· P D.
54
23. Si desde un punto P se trazan dos semirrectas con los puntos A, B sobre una y los puntos
C, D sobre la otra, tales que P A· P B = P C· P D, entonces los puntos A, B, C, D están
sobre una circunferencia.
24. Si desde un punto exterior a una circunferencia se trazan dos semirrectas, una tangente
y la otra secante, entonces el segmento entre el punto y el punto de tangencia es media
proporcional entre los segmentos determinados entre el punto exterior y los puntos de
intersección de la secante con la circunferencia. 20
25. Si P es un punto sobre el mismo plano que una circunferencia de centro O y radio r, y d
es la distancia del punto P al centro O de la circunferencia, demuestre que:
a) Si P está en el interior de la circunferencia, entonces la potencia de P es r2 − d2 .
b) Si P está en el exterior de la circunferencia, entonces la potencia de P es d2 − r2 .
c) Si P está sobre de la circunferencia, entonces la potencia de P es cero.
26. (*) (IV OMCC, P-4) Sea ABC un triángulo, D el punto medio de BC, E un punto sobre
el segmento AC tal que BE = 2AD y F el punto de intersección de AD con BE. Si
∠CAD = 60°, encuentre la medida de los ángulos del 4F EA.
27. (*) Sea ABCD es un trapecio con AD k BC. M y N son los puntos medios de CD y BC,
M
= 14 , demuestre que
respectivamente, y P el punto común de las rectas AM y DN . Si PAP
ABCD es paralelogramo.
28. Dado el 4ABC se construye un cuadrado P QRS con P en AB, Q en AC, R y S en BC.
Sea H el pie de la altura desde A hacia BC. Demuestre que:
1
1
1
=
+
PQ
AH BC
b) (ABC) = 2(P QRS) si y sólo si AH = BC.
a)
29. Sea P un punto en el interior del 4ABC. Se trazan por P las paralelas a los lados del
triángulo, que queda dividido en tres triángulos y tres paralelogramos. Si las áreas de los
tres triángulos de la subdivisión son, en algún orden, 9, 16 y 25, hallar el área del 4ABC.
Problemas de Refuerzo.
30. (*) En la figura anexa, BC = CD = DE =
EA = x y ∠AEB = 90°. Demuestre que
∠ABC + ∠ACD + ∠ADE = 90°.
20
Los problemas anteriores nos permite establecer la siguiente definición de Potencia de un punto con
respecto a una circunferencia: La potencia de un punto P con respecto a una circunferencia de centro O y
radio r es el producto P A· P B, donde A y B son los puntos de intersección de la circunferencia con una recta
que pasa por P .
55
31. Las tres circunferencias de la figura 80 tienen el mismo radio r, sus centros son colineales
y la circunferencia de centro O2 es tangente a las otras dos. Por A se traza una tangente
a la circunferencia de centro O3 . Obtenga el valor del segmento BC en función de r.
Figura 80
32. Sea ABCD un rombo, con AC = 6 y BD = 8. Se construyen exteriormente los cuadrados
ADEF y CDHG, cuyos centros son O1 y O2 , respectivamente (Vea figura 81). Calcular
la medida del segmento O1 O2 .
Figura 81
33. Sea ABCD un cuadrado con P y Q sobre AB y BC tales que BP = BQ. Sea H el pie
de la perpendicular de B a P C. Demuestre que DHQ = 90°.
34. Dado un paralelogramo ABCD, se trazan dos circunferencias tangentes externamente
en P , y tales que la primera es tangente internamente al ∠ABC y la otra es tangente
internamente al ∠CDA, como en la figura 82. Demuestre que B, P y D están alineados.
56
Figura 82
35. En un 4ABC el ∠CAB = 120. Encuentre la medida de la bisectriz interna del ∠CAB
en función de los lados adyacentes.
36. El 4ABC tiene lados de 13, 14 y 15 unidades. El 4A0 B 0 C 0 está dentro del 4ABC con
lados paralelos a los de éste y a 2 unidades de distancia de los lados del mismo. Calcule
(ABC) − (A0 B 0 C 0 ).
37. (*) (Asiático Pacı́fica) Sea ABC un triángulo y D el pie de la altura con respecto a A.
Sean E y F puntos en una recta que pasa por D (distintos de D) tales que AE ⊥ CE
y AF ⊥ BF . Sean M y N los puntos medios de BC y EF , respectivamente. Demuestre
que AN ⊥ N M .
57
7.
Puntos y Rectas Notables del Triángulo.
MEDIANAS
Definición: En un triángulo, una mediana es el segmento de recta que une un vértice con el
punto medio del lado opuesto.
Teorema: Las tres medianas de un triángulo concurren en un punto llamado el Centroide 21
del triángulo y usualmente es denotado por G. Además, las medianas de cortan mutuamente
en razón 2:1.
Demostración: Dado el 4ABC sean A0 , B 0 , C 0 , los puntos medios de BC, CA, AB, respectivamente. Defina G como la intersección de BB 0 con CC 0 . Por el teorema de la base media,
B 0 C 0 k BC y 2B 0 C 0 = BC; observe que 4BCG ' B 0 C 0 G, con razón de semejanza 2, por lo
que
GC
GB
=
=2
0
GB
GC 0
Análogamente, si G∗ = AA0 ∩ BB 0 se cumple
G∗ A
G∗ B
=
=2
G∗ B 0
G∗ A0
Ası́, G y G∗ dividen al segmento BB 0 en dos segmentos cuya razón es 2:1, por lo que G = G∗ ,
lo cual implica que AA0 , BB 0 , CC 0 concurren y
GB
GC
GA
=
=
=2
0
0
GA
GB
GC 0
MEDIATRICES
Definición: Dado un segmento AB, la mediatriz del segmento es el lugar geométrico de puntos
que equidistan de A y B, i.e., un punto P está sobre la mediatriz de AB si y sólo si P A = P B.
Teorema: La mediatriz de AB es una recta l perpendicular a AB y que pasa por su punto
medio.
Demostración: Sea M el punto medio de AB, y l pasa por M y l ⊥ AB. En primer lugar
se probará que todos los puntos de l satisfacen la definición de mediatriz: Por definición de
punto medio M A = M B. por lo que claramente M pertenece a la mediatriz de AB; sea P un
punto de l distinto de M , por criterio LAL, 4P M A ≡ 4P M B por lo que P A = P B. Ahora,
cabe preguntarse si existe algún punto fuera de l que también cumpla la definición: suponga
P 0 tal que P 0 A = P 0 B, esto implica que 4P 0 AB es isósceles, y entonces ∠P 0 AB = ∠P 0 BA;
si M 0 es la proyección de P 0 sobre AB, por criterio ALA 4P 0 AM 0 ≡ 4P 0 BM 0 , lo cual implica que M 0 A = M 0 B, es decir que M 0 = M , y esto obliga a que P 0 esté sobre l (ya que P 0 M 0 = l).
Teorema: Las mediatrices de un 4ABC concurren en un punto que equidista de los vértices
del triángulo, llamado el Circuncentro del 4ABC
21
También conocido como Geocentro, Centro de Gravedad, Baricentro, o más formalmente Equibaricentro.
58
Figura 83: Concurrencia de Mediatrices, Circuncentro y Circuncı́rculo.
Usualmente, el circuncentro es denotado por O, y R representa la distancia del circuncentro a
los vértices
R = OA = OB = OC
A esta distancia se le llama Circunradio del 4ABC. Ası́, O es el centro de una circunferencia
que pasa por A, B, C, cuyo radio es R, llamada el Circuncı́rculo del 4ABC.22
Demostración: Sea O la intersección de las mediatrices de AB y BC, por el teorema anterior,
como O pertenece a la mediatriz de AB se cumple OA = OB, y como también pertenece a la
mediatriz de BC, OB = OC; entonces OC = OA, y utilizando de nuevo el teorema anterior,
O debe pertenecer a la mediatriz de CA. Ası́, las tres mediatrices concurren en O, y este punto
equidista de los vértices del 4ABC.
Corolario: Dado un triángulo, existe una circunferencia que pasa por los tres vértices (el circuncı́rculo); además, esta circunferencia es única.
Una observación importante es que la mediatriz del lado de un triángulo NO siempre pasa por
el vértice opuesto; de hecho, esto sólo se da si el triángulo es isósceles.
ALTURAS
La altura es un concepto que está intrı́nsecamente relacionado con la distancia de un punto a
una recta; la altura es la recta que debe trazarse para determinar esta distancia, i.e., es una
recta que pasa por el punto y es perpendicular a la recta. A la intersección entre la altura y la
recta generalmente se le llama pie de la altura, o también (más formal) proyección del punto
sobre la recta. En particular, para triángulos, definiremos la altura de la siguiente forma:
Definición: Dado un triángulo, una altura es una recta que pasa por un vértice y es perpendicular al lado opuesto.
22
En ocasiones, denotaremos a esta circunferencia por Γ(ABC).
59
Es importante observar que el pie de la altura NO siempre pertenece a un lado; de hecho, una
altura puede estar “al interior” de un triángulo, coincidir con un lado, o estar completamente
afuera de un triángulo.
Teorema: Las alturas de un triángulo concurren en un punto, llamado el Ortocentro del triángulo, usualmente denotado por H.23
Demostración: Dado el 4ABC, se construyen los puntos A1 , B1 , C1 , tales que ABA1 C,
BCB1 A, CAC1 B son paralelogramos. Observe que el 4ABC es el triángulo medial del 4A1 B1 C1 ,
y que las alturas del 4ABC son las mediatrices del 4A1 B1 C1 ; como las mediatrices de un
triángulo concurren (en este caso, las del 4A1 B1 C1 ), las alturas del 4ABC concurren.
La altura también puede escribirse en términos de lugar geométrico:
Teorema: La recta l es perpendicular a AB si y sólo si AL2 − LB 2 es constante. Es decir, que
una recta perpendicular a AB es el lugar geométrico de los puntos L que satisfacen la condición
anterior.
Demostración: sea P la intersección de l con AB, y L un punto arbitrario sobre l; por Pitágoras se tiene AL2 − LB 2 = AP 2 − P B 2 , y el término derecho de la igualdad es constante. La
otra dirección de la implicación se prueba por contradicción.
De esa definición también puede fabricarsele una demostración del teorema anterior, sin embargo, no se aborda porque la prueba se basa en un resultado sofisticado llamadado Teorema
de Steiner.24
BISECTRICES
Definición: La bisectriz de un ángulo es una recta que “divide” al ángulo en dos ángulos de
igual magnitud.
Teorema: El lugar geométrico de puntos que equidistan de dos rectas dadas, generan un par
de rectas perpendiculares llamadas bisectriz interna y bisectriz externa del ángulo formado por
las rectas.
Demostración: Suponga que las rectas se cortan en un punto O; sean a, b las rectas dadas, y
P un punto que equidista de ellas; si A y B son las proyecciones de P sobre a y b, respectivamente, entonces P A = P B. Observe que por criterio LLL (utilizando Pitágoras previamente),
4OAP ≡ 4OBP , por lo que ∠P OA = ∠P OB, i.e., P pertenece a la bisectriz del ∠AOB.
Claramente aquı́ se dan dos casos, recuerde que para definir el ángulo entre a y b se utilizan únicamente semi-rectas, por lo que las rectas a y b definen cuatro ángulos, que por parejas pueden
23
El triángulo formado por los pies de las alturas de un 4ABC es llamado el triángulo órtico del 4ABC.
Sean l, m, n, tres rectas perpendiculares a los lados del AB, BC, CA del 4ABC, respectivamente. Sean
L, M , N , puntos arbitrarios sobre l, m, n, respectivamente. Entonces las rectas l, m, n concurren si y sólo si
AL2 + BM 2 + CN 2 = N A2 + LB 2 + M C 2 .
24
60
ser opuestos por el vértice o suplementarios; de estos se escoge cualquiera de ellos como referencia, entonces, si ∠AOB coincide con éste o con el opuesto por el vértice, la recta P O es llamada
bisectriz interna, y en caso contrario, bisectriz externa. Ası́, el lugar geométrico son dos rectas,
y su perpendicularidad se basa en los pares de ángulos que son suplementarios. Finalmente, si
a k b, el lugar geométrico es una recta paralela a a y b que se ubica entre ellas a igual distancia de
ambas (este es un caso extraño de bisectriz interna, sin embargo, en ocasiones es útil tener esta
convención en mente; peor aún, la bisectriz externa es una recta ideal llamada recta al infinito).
Teorema: Las bisectrices internas de un 4ABC concurren en un punto, llamado el Incentro
del 4ABC, usualmente denotado por I. La distancia de I a los tres lados del triángulo es igual
a un número r, llamado el Inradio del 4ABC, y de aquı́ que la circunferencia de centro I y
radio r sea tangente a los lados del triángulo; dicha circunferencia es llamada el Incı́rculo del
4ABC.25
Figura 84: Concurrencia de Bisectrices Internas, Incentro e Incı́rculo.
Demostración Sea I la intersección de las bisectrices internas de ∠A y ∠B (obviamente, I
está en el interior del 4ABC); como I pertenece a la bisectriz interna del ∠A, por el teorema
anterior dist(I, AB) = dist(I, AC), y análogamente, como I pertenece a la bisectriz interna del
∠B, dist(I, AB) = dist(I, CB); entonces dist(I, AC) = dist(I, CB), y de nuevo por el teorema
anterior y dado que I está al interior del triángulo, I pertenece a la bisectriz interna del ∠C.
Ası́, las tres bisectrices internas concurren en un punto que equidista de los lados del triángulo.
Es importante notar que las intersección de una bisectrices interna con el lado opueto del
triángulo NO siempre coincide con el puntos de tangencia del incı́rculo;26 de hecho, esto ocurre
sólamente si el triángulo es isósceles.
Corolario: Dado un triángulo, existe una circunferencia que es tangente interiormente a los
tres lados (el incı́rculo); además, esta circunferencia es única.27
25
En algunas ocasiones denotaremos al incı́rculo por Λ(ABC).
En la figura, el 4ABC es llamado triángulo tangencial del 4DEF .
27
Existen 3 circunferencias más que son tangentes a los tres lados del triángulo, llamados excı́rculos; estas
circunferencias se ubican en el exterior del triángulo.
26
61
Ejercicios
1. Las áreas de los seis triángulos AGB 0 , AGC 0 , BGA0 , BGC 0 , CGA0 , CGB 0 son iguales e
iguales a un 61 del área del triángulo ABC.
Figura 85
2. Los cuatro triángulos AB 0 C 0 , BC 0 A0 , CA0 B 0 , A0 B 0 C 0 ,28 son congruentes entre si y semejantes al 4ABC con razón de semejanza 12 .
3. El centroide del 4ABC coincide con el centroide del triángulo medial 4A0 B 0 C 0 . Además,
estos dos triángulos tienen lados correspondientes paralelos (triángulos homotéticos).
4. En la figura 86, G es el centroide. Si GD = 2 y el área sombreada vale 5, calcule AD y el
(ABC).
Figura 86
5. Demostrar que las paralelas a los lados de un 4ABC, trazadas por el centroide G dividen
cada lado en tres partes iguales.
6. ABCD es un paralelogramo de centroide (baricentro) E, M es el punto medio de AD,
y F es la intersección de AC con BM . Si el área de ABCD es 1, calcule el área del
cuadrilátero DEF M .
7. En el 4ABC, se traza la mediana AM . Demostrar que si BM = AM , entonces el
triángulo es rectángulo en A.
28
El 4A0 B 0 C 0 es llamado el triángulo medial del 4ABC.
62
8. La suma de las distancias del centroide a los puntos medios de los lados de un triángulo
es 20. Calcule la suma de las medianas del triángulo.
9. La mediana tiene longitud menor que la semisuma de los lados adyacentes, es decir AA0 <
b + c, BB 0 < c + a, CC 0 < a + b.
10. Dado el 4ABC, sean D y E puntos variables sobre los lados AB y AC respectivamente
tales que BC k DE. Entonces, la mediana AA0 puede definirse como el lugar geométrico
de los puntos P tales que P ∈ CD ∩ BE.29
11. Siempre es posible construir un triángulo XY Z con las medianas AA0 , BB 0 , CC 0 de un
4ABC dado. Además, los segmentos que unen el centroide del 4XY Z con sus vértices
son iguales a la mitad de los lados del 4ABC.
12. En el 4ABC, AB = BC y la mediatriz de BC interseca a la mediana BM en L. Si
∠LCB = 25, determine la medida del ∠LAC.
13. Ley del Seno. Dado un 4ABC, se cumple que
sen∠B
sen∠C
1
sen∠A
=
=
=
a
b
c
2R
14. Las reflexiones de H con respecto a los lados del 4ABC caen sobre el circuncı́rculo del
mismo, es decir HHa = Ha X y análogo para los otros lados.
Figura 87
15. Las reflexiones de H con respecto a los puntos medios de los lados del triángulo, caen
sobre el circuncı́rculo del mismo.
29
Si D = A se define P = A, y cuando D = B entonces P es punto medio de BC.
63
16. Si O y H son el circuncentro y el ortocentro de un 4ABC, respectivamente, entonces
∠BAH = ∠CAO.
17. La altura AHa es bisectriz del ∠Hb Ha Hc .
18. Los circuncı́rculos de 4ABC, 4ABH, 4BCH, 4CAH tienen igual radio.
19. La perpendicular trazada desde A al lado Hb Hc del triángulo órtico, pasa por el circuncentro del 4ABC.
20. A, B, C y H forman un cuadrilátero ortocéntrico, es decir que cada punto es el ortocentro
del triángulo formado por los otros tres.
21. El ortocentro de un triángulo está al interior, sobre un vértice, o afuera del triángulo, si
el triángulo es acutángulo, rectángulo, u obtusángulo, respectivamente.
22. El circuncentro del 4ABC es el ortocentro del triángulo medial 4A0 B 0 C 0 .
23. Sea O el circuncentro del 4ABC. Si ∠AOC = 100 y ∠OCB = 30, determine la medida
de los ángulos del 4ABC.
24. Hallar los ángulos de un triángulo cuyo triángulo órtico tiene ángulos de 20, 50 y 110.
25. Sea ABC un triángulo obtusángulo de circuncentro O y altura AD. Si ∠OAB = 25 y
∠OCB = 15, calcule el ∠DAB.
26. El 4ABC de circuncentro O y altura BD. Si ∠DAB = 35 y ∠OBD = 10 encontrar los
ángulos del triángulo ABC.
27. En la figura 88, AB es diámetro de la circunferencia. Si X es la intersección de CG con
AB, calcular el ∠CXB.
Figura 88
64
28. En el 4ABC, se trazan la altura AH y la mediana BM . Demuestre que el 4M HC es
isósceles.
29. Un 4ABC es rectángulo en C, ∠A = 75 y CH es altura. Demuestre que CH =
AB
.
4
30. Sea O el circuncentro del 4ABC con ∠C = 45 y sea D el pie de la altura desde A.
Calcule la medida del ∠ODC.
31. Dado el 4ABC isósceles con ∠A = 90, sean P y Q son puntos dentro del triángulo tales
que BP = AQ y AP = CQ. Si BP y CQ se cortan en R, demostrar que AR ⊥ P Q.
32. Se ubican los puntos M y K sobre los lados BC y CD del cuadrado ABCD, respectivamente, de modo que M C = KD. Sea P la intersección de M D y BK, demuestre que
AP ⊥ M K.
33. Sean D, E, F los puntos de tangencia del incı́rculo sobre los lados BC, CA, AB del
4ABC. Demuestre que se cumplen las siguientes relaciones, donde s denota el semiperı́metro del triángulo:
AE = AF = s − a
BD = BF = s − b
CD = CE = s − c
34. El ortocentro del 4ABC es el incentro de su triángulo órtico.
35. Dado un 4ABC, su triángulo órtico y su triángulo tangencial tienen lados correspondientes paralelos (triángulos homotéticos).
36. Las bisectrices exteriores de ∠B y ∠C, junto con la bisectriz interior de ∠A, concurren
en un punto, llamado el Excentro con respecto al vértice A, usualmente denotado por Ia .
Este punto es equidistante a los lados del 4ABC, dicha distancia es el Exradio respecto a
A, usualmente denotado por ra . Ası́, la circunferencia de centro Ia y radio ra es tangente
exteriormente a los lados del 4ABC, y es llamada el Excı́rculo respecto a A.30
37. I es ortocentro del 4Ia Ib Ic . Además se cumple:
AX = AZ = s
BX = BY = s − b
CY = CZ = s − c
38. En un 4ABC, la bisectriz exterior del ∠ABC y la bisectriz exterior del ∠BCA se cortan
en D. La paralela a BC por D corta a AC en L y a AB en M . Si LC = 5 y M B = 7,
hallar LM .
39. El 4ABC es rectángulo en A. Si I es el incentro, calcular ∠BIC.
40. En un 4ABC, el ∠ABC − ∠CAB = 90. Sean D y E los pies de las bisectrices interior
y exterior del ∠BCA respectivamente. Demuestre que CD = CE.
30
Análogamente se definene los excı́rculos con respecto a los otros vértices.
65
Figura 89
41. En el 4ABC, AB < AC, AD es bisectriz, y E es un punto en AB tal que el ∠EDB = 90.
El punto F sobre AC es tal que el ∠BED = ∠DEF . Demuestre que el ∠BAD = ∠F DC.
42. En el 4ABC se trazan las bisectrices interiores BD y CE tales que D es el punto sobre
AC, E es el punto sobre AB, 2∠BDE = 3∠B y ∠CED = 2∠B. Calcular los ángulos del
4ABC.
43. Dado el 4ABC con ∠A = 90, sea D el pie de la perpendicular desde A. Sean además I
y J los incentros respectivos de 4ABD y 4ACD. Demostrar que la bisectriz del ∠BAC
es perpendicular a IJ.
44. Un triángulo es isósceles si cumple alguna de las siguientes condiciones:
a) Dos medianas son iguales.
b) Dos alturas son iguales.
c) Dos bisectrices son iguales.31
45. Teorema de la Bisectriz: Dado el 4ABC, sean P y P 0 sobre BC. Se cumple que AP
y AP 0 son la bisectriz interna y la bisectriz externa del ∠A si y sólo si
BA
BP 0
BP
=
= 0
PC
AC
PC
Sugerencia: Para demostrar la primera igualdad, trace CD k AP con E sobre la prolongación de AB.
31
Este caso es aparentemente tan sencillo como los anteriores, pero realmente es un resultado muy complicado
y recibe el nombre de Teorema de Steiner-Lehmus.
66
46. (*) De acuerdo con los datos de la gráfica 90, calcular el valor de AB.
Figura 90
47. Dos circunferencias son tangentes internamente en P , y una cuerda AB de la circunferencia de radio mayor es tangente en Q a la otra circunferencia. Ver figura 91.
a) Demuestre que P Q es bisectriz del ∠AP B.
b) Llame A0 y B 0 a las otras intersecciones de P A y P B con la circunferencia de radio
menor y suponga que AB = 15, P A0 = 3 y P B 0 = 2; calcule AQ y BQ.
Figura 91
48. (*) Sea ABC un triángulo tal que las medianas respectivas a B y C son perpendiculares.
Demuestre que se cumple la relación.
5BC 2 = CA2 + AB 2 .
49. (*) Teorema de Poncelet: Demuestre si 4ABC es un triángulo rectángulo con ∠A =
90°, entonces 2(r + R) = b + c.
67
Problemas de Refuerzo.
50. (*) Sea ABCD un paralelogramo. Q es el punto medio de AD, F el pie de la perpendicular
por B sobre QC. Probar que AF = AB.
51. Dado el rombo ABCD, se trazan las bisectrices internas de ∠DAC, ∠CAB, ∠BCA,
∠ACD, y cortan a DC, CB, BA, AD en P , Q, R, S, respectivamente. Demuestre que
P QRS es un rectángulo.
52. (*) Sea ABCD un cuadrilátero tal que AB = CD. Las mediatrices de AC y BD se cortan
en P . Probar que ∠P AC = ∠P CA = ∠P BD = ∠P DB.
53. (*) ABC es un triángulo y P un punto en su interior. Sean A0 , B 0 y C 0 las reflexiones de
P sobre BC, CA y AB, respectivamente. D, E y F son los pies de las perpendiculares
respectivos desde A, B y C hacia B 0 C 0 , C 0 A0 y A0 B 0 . Probar que AD, BE y CF son
concurrentes.
54. (*) (Arnoldo Aguilar) En la figura 92, ABGH, BCF G y CDEF son cuadrados. Si I es
el centro de ABGH y J = DH ∩ BG, demuestre que I, J y F están alineados.
Figura 92
55. (*) (Arnoldo Aguilar) Sea ABC un triángulo equilátero. M y N son los puntos medios de
AB y BC, respectivamente. Exteriormente al 4ABC se construye un triángulo rectángulo
isósceles 4AP C, con ∠AP C = 90◦ . Si I es la intersección de AN y M P , demuestre que
CI es la bisectriz de ∠ACM .
56. (*) En la figura adjunta, el 4ABC es tal que ∠A = 90° y
∠B = 60°. ¿Cuál es el radio de la circunferencia?
57. (*) Dado el paralelogramo ABCD, sea M el punto medio de AB, y N la intersección de
CD con la bisectriz interna del ∠ABC. Demuestre que M C ⊥ BN si y sólo si AN es
bisectriz del ∠DAB.
68
58. (*) En el 4ABC, se sabe que los vértices B, C, el circuncentro O y el ortocentro H del
4ABC están todos sobre una misma circunferencia.
a) Calcule el valor de ∠A.
b) Demuestre que el incentro también pertenece al circuncı́rculo de BCOH.
59. (*) Sea ABC un triángulo de ortocentro H. Sean P y Q los pies de las perpendiculares
desde H a las bisectrices interior y exterior de A, respectivamente. Si M es el punto medio
de BC, mostrar que P , Q y M están alineados.
60. (*) En un triángulo ABC, sea M el punto medio de BC. Si se cumple que AB 6= AC y
además ∠M AC + ∠ABC = 90◦ , hallar ∠BAC.
61. (*) Sea ABC un triángulo y U un punto de su circuncı́rculo tal que AU es bisectriz. Las
mediatrices en AB y AC cortan a AU en X y Y . Sea T la intersección de BX con CY .
Demostrar que AU = T B + T C.
62. (*) (The 59th Romanian Mathematical Olympiad Final Round) Sea ABCD un rectángulo
de centro O con AB 6= BC. La perpendicular en O a BD corta a las lı́neas AB y BC en
los puntos E y F , respectivamente. Sean M y N los puntos medios de los segmentos CD
y DA, respectivamente. Probar que las lı́neas rectas F M ⊥ EN .
63. (*) Sea ABC un triángulo rectángulo, con A = 90◦ . Sea D un punto en su interior tal
que ∠DAC = ∠DCA = ∠DBC = α, y AC = BD. Determine el valor de α.
64. (*) Sea ABC un triángulo y M un punto tal que ∠M AB = 10, ∠M BA = 20, ∠M AC =
40 y ∠M CA = 30. Probar que el 4ABC es isósceles.
65. (*) En la figura 93, ABCD y P QRS son cuadrados, 4ABP ≡ 4BCQ ≡ 4CDR ≡
4DAS y los los radios de las cinco circunferencias son iguales a r. Si a es el lado del
cuadrado ABCD, determine r en función de a.
Figura 93
69
66. (*) Recta de Euler. El centroide G, el ortocentro H y el circuncentro O de un triángulo
están alineados, y además GH = 2GO.
67. Circunferencia de los 9 puntos:32 Dado un 4ABC de ortocentro H, se cumple que
los puntos medios de los lados, los pies de las alturas, y los puntos medios de HA, HB,
HC, se ubican sobre una misma circunferencia. Además, el centro de esta circunferencia
es el punto medio de HO y su radio es R2 , donde O y R son el circuncentro y el circunradio
del triángulo.
Para demostrar este resultado se sugiere seguir los siguientes pasos:
a) Si Ha es el pie de la altura trazada desde A, demuestre que la reflexión de H con
respecto a Ha pertenece a circuncı́rculo del 4ABC. Resultados similares se cumplen
para Hb y Hc .
b) Si A0 es el punto medio de BC, demuestre que la reflexión de H con respecto a A0
pertenece al circuncı́rculo del 4ABC.
c) De los resultados anteriores, observe que hay 9 puntos sobre el circuncı́rculo del
4ABC: los vértices, las reflexiones de H con respecto a los pies de las alturas, y
las reflexiones de H con respecto a los puntos medios de los lados; a partir de esto,
concluya que los puntos medios de los segmentos que van de H a estos 9 puntos,
también deben pertenecer en una misma circunferencia.
d) Concluya además que el centro de esta nueva circunferencia es el punto medio de HO.
Otro camino de solución es el siguiente:
a) Sea 4A0 B 0 C 0 el triángulo medial del 4ABC. Pruebe que ∠A0 B 0 C 0 = ∠BHa C 0 y
concluya que Ha A0 B 0 C 0 es un cuadrilátero cı́clico; los mismo debe cumplirse para Hb
y Hc .
b) Sea X el punto medio de HA. Demuestre que ∠B 0 A0 C 0 +∠B 0 XC 0 = 180 y concluya que
XC 0 B 0 A0 es un cuadrilátero cı́clico; lo mismo debe cumplirse para los puntos medios
de HB y HC.
c) De lo anterior, concluya que los pies de las alturas y los puntos medios de los segmentos
que van desde H hasta los vértices del 4ABC, se ubican sobre el circuncı́rculo del
4A0 B 0 C 0 .
d) Si N es el circuncentro del 4A0 B 0 C 0 , demuestre que N , O, G forman la recta de Euler
del 4A0 B 0 C 0 y utilice sus propiedades para probar que N es el punto medio de HO.
68. El área del 4ABC, denotada por [ABC], cumple:
base × altura
2
ab sen∠C
bc sen∠A
ca sen∠B
abc
[ABC] =
=
=
=
2
2
2
4R
[ABC] = sr
p
[ABC] =
s(s − a)(s − b)(s − c). (Fórmula de Herón).
[ABC] =
32
También conocida como Circunferencia de Feuerbach.
70
69. El circunradio, el inradio y los exradios de un triángulo cumplen:
4R = ra + rb + rc − r
[ABC] = ra (s − a) = rb (s − b) = rc (s − c)
r
(s − a)(s − b)(s − c)
r =
s
r
s(s − b)(s − c)
ra =
s−a
r
s(s − a)(s − c)
rb =
s−b
s
s(s − a)(s − b)
rc =
(s − c)
70. Dado el 4ABC, sea I el incentro e Ia el excentro respecto a A.
a) Demuestre que BICIa es un cuadrilátero cı́clico.
b) Si M es la intersección de IIa con el circuncı́rculo del triángulo (M 6= A), demuestre
que dicho punto es el circuncentro de BICIa .
c) Sea M 0 el punto diametralmente opuesto a M en el circuncı́rculo, y sea P la proyección
de I sobre AB. Demuestre que 4M 0 CM ' 4AIP .
d) Sea O el circuncentro del triángulo Calcule la potencia de punto de I con respecto
al circuncı́rculo, y utilizando los resultados anteriores deduzca la Fórmula de Euler :
OI 2 = R2 − 2Rr.
e) A partir de la fórmula de Euler demuestre que R ≥ 2r.
71
8.
Solución a Problemas Selectos.
Teoremas Fundamentales del Triángulo.
1. En la figura 94, ABDE es un cuadrado y BCD es un triángulo isósceles con BD = DC.
Si ∠ABC = 160, determinar la medida de ∠AEC.
Figura 94
Solución: ∠DBC = ∠DCB = 160 − ∠ABD = 70, de donde se obtiene que ∠BDC = 40
y ∠EDC = ∠EDB + ∠BDC = 130. Como 4EDC es isósceles, entonces ∠DEC =
∠DCE = 25. Por lo tanto ∠AEC = 90 − ∠DEC = 65.
2. Hallar la suma de los ángulos α + + θ + φ en la figura 95.
Figura 95
Solución: ∠CAB = θ, ∠EDC = α por ser opuestos por el vértice. Como el ∠AF D
externo en el 4BDF , se tiene ∠AF D = α + . Sumando los ángulos internos del 4AEF
se tiene α + φ + θ + = 180°.
3. (XV Competencia de Clubes Cabri Primera Ronda) En la figura 96, ABCD es un
rectángulo tal que AB = 2BC. M es el punto medio de AB y los triángulos AM E
y M BF son equiláteros. Si P es la intersección de las rectas DE y CF , encuentre los
ángulos del 4CDP .
72
Figura 96
Solución: Note que AD = AE = F B = BC por lo que 4DAE y 4BCF son ambos
isósceles. Luego ∠DAE = ∠CBF = 90+60 = 150 lo que implica que ∠P DC = ∠P CD =
90 − 15 = 75 y luego ∠CP D = 30.
4. Sea ABC un triángulo rectángulo con ∠CAB = 90 (Ver figura 97). D es un punto sobre
la prolongación de BC tal que BD = BA. E es un punto en el mismo semiplano que A
respecto de BC, tal que CE ⊥ BC y además CE = CA. Mostrar que A, D y E están
alineados.
Figura 97
Solución: Sea ∠CBA = 2θ; el 4ABD es isósceles y ∠BAD + ∠BDA = 2θ, por lo
que ∠BAD = ∠BDA = θ. Como ∠CAB = 90 entonces ∠ACB + ∠ABC = 90 y como
CE es perpendicular a BC entonces ∠ECA + ∠ACB = 90; por lo tanto, ∠ABC =
∠ECA = 2θ. Con esto, como 4ECA es isósceles, ∠CEA = ∠CAE = 90 − θ. Luego,
∠EAC + ∠CAB + ∠BAD = 180 y ası́ E, A y D están alineados.
5. Dado un cuadrado ABCD, se construyen los triángulos equiláteros ABP (exteriormente)
y ADQ (interiormente). Probar que C, P y Q están alineados. Figura 98.
73
Solución: Observe que ∠P AQ = ∠BAD = 90 y P A = BA = DA = DQ, por lo
que 4P AQ es triángulo rectángulo isósceles, y por tanto, ∠P QA = 45. Por otra parte,
∠QDC = 90 − ∠ADQ = 30 y QD = AD = CD, es decir, el 4CDQ es isósceles
con el ángulo comprendido entre lados iguales de 30, por lo que ∠DQC = 75. Ası́,
∠P QA + ∠AQD + ∠DQC = 180 y por lo tanto, C, P y Q están alineados.
Figura 98
6. En la figura 99, AB = BC = CD = DE = EF = F G = GA. Calcule la medida del
∠DAE. Referenciasfig20
Figura 99
Solución: Sea ∠DAE = θ. Como los triángulos ABC y AGF son isósceles, ∠ACB =
∠AF G = θ. Calculando los ángulos externos de 4ABC y 4AF G se tiene ∠F BC =
∠CGF = 2θ. Como 4GF E y 4BCD son isósceles, ∠GEF = ∠BDC = 2θ. Calculando
ángulos externos de 4ADC y 4AEF se obtiene ∠ECD = ∠DF E = 3θ. Como 4CDE
y 4F ED son isósceles, ∠CED = ∠F DE = 3θ. Entonces, la suma de los ángulos internos
del 4AED da θ + 3θ + 3θ = 180, de donde θ = 180
.
7
7. (XXVIII Olimpiada Brasileña de Matemática) En la figura 100, AB = AC, AM = AN
y ∠CAM = 30°, encuentre el valor del ∠BM N .
Solución: Como 4ABC y 4AM N son isósceles, sean ∠ABC = ∠ACB = α y ∠AM N =
∠AN M . Por la fórmula del ángulo externo se tiene
74
Figura 100
∠ACM + ∠M AC = ∠AM B = ∠AM N + ∠BM N
α + 30
=
∠AN M + ∠BM N
=
(∠N BM + ∠BM N ) + ∠BM N
=
α + 2∠BM N
Esto implica que ∠BM N = 15°.
8. (Etapa semifinal Estatal de XXII Olimpiada Mexicana de Matemáticas) En la figura
101 se muestra un hexágono regular ABCDEF de lado 1. Los arcos del cı́rculo que están
dibujados tienen centro en cada vértice del hexágono y radio igual a la distancia al vértice
opuesto. P , Q, R, S, T y U son los puntos de corte de estos arcos. ¿Cuánto mide cada
lado del hexágono P QRST U ?
Figura 101
Solución: El hexágono P QRST U es regular y con el mismo centro que ABCDEF . Sea
O el centro de ambos (Véase Figura 64). El lado buscado es igual a OP . Tenemos que
CF = F P = P C = 2 por ser radios de los arcos dibujados; entonces CF P es√equilátero
de lado 2 y OP es una altura de este triángulo que, por Pitágoras, es igual a 3.
75
Congruencia de Triángulos
1. En la figura 102, ABC es un triángulo equilátero y CDEF es un cuadrado. Se construye
un punto G tal que CF = CG y además ∠CF G = 15°. Probar que ∠AGC = ∠BDC.
Figura 102
Solución: ∠BCD = 180 − ∠ACB − ∠DCF = 30. Como 4GCF es isósceles, ∠CGF =
∠CF G = 15 y ∠ACG = ∠CGF + ∠CF G = 30. Por criterio LAL, 4BCD ≡ 4ACG,
por lo tanto ∠BDC = ∠AGC.
2. (Cuaderno de Olimpiadas Mexicanas - Geometrı́a) En la figura 103, ABCD un cuadrado
y EF ⊥ GH. Demuestre que que EF = GH.
Figura 103
Solución: Se construyen EK y GM con K sobre CD y M sobre AD tales que EK k AD
y GM k CD. Luego se demuestra que 4EF K ≡ 4GHM , con EF = GH.
3. (Examen final de XVI Olimpiada mexicana de Matemática) Los ángulos de un triángulo
ABC están en progresión aritmética (∠B − ∠A = ∠C − ∠B = θ), D, E, y F son los
puntos medios de los lados BC, CA y AB, respectivamente. Llamamos H al pie de la
altura trazada desde C (que cae entre B y F ) y G a la intersección entre DH y EF .
¿Cuánto vale ∠F GH?
Solución: Note que ∠A + ∠B + ∠C = 3∠A + 3θ = 180, lo cual implica que ∠A + θ =
76
60 = ∠B. Entonces 4BCH es un triángulo 30, 60, 90, y dado que D es punto medio de
BC, el 4BDH es equilátero. Luego, como BC k EG, ∠F GH = ∠BDH = 60. Ver figura
104.
Figura 104
4. Sea ABCD un cuadrado. Se construyen triángulos equiláteros ADP y ABQ como se
muestra en la figura 105. Sea M la intersección de CQ con AD y N la intersección de
CP con AB. Demuestre que CM N es un triángulo equilátero.
Figura 105
Solución: Note que P D = AD, porque 4AP D es equilátero, y AD = CD porque
ABCD es cuadrado, por lo que P D = CD, es decir, el 4CDP es isósceles, con ∠CDP =
∠CDA − ∠P DA = 30, entonces ∠DP C = ∠DCP = 75, y ∠BCN = ∠BCD − ∠DCP =
15. Análogamente, 4BCQ es isósceles con ángulos 30, 75, 75, por lo que ∠M CN =
∠BCQ − ∠BCN = 60. Finalmente, como la figura es simétrica con respecto a AC,
CM = CN , entonces, el triángulo CM N es equilátero porque tiene dos lados iguales y
un ángulo interno igual a 60.
5. 4ABC es un triángulo isósceles con ∠ABC = ∠ACB = 80°. D es un punto en AC tal
que ∠ABD = 10°. Demuestre que AD = BC. Figura 106.
77
Solución: Se traza un punto D0 sobre AC tal que AD0 = BC. Se construye exteriormente
el triángulo equilátero AEB. Luego, AE = AB, D0 A = CB y ∠EAD0 = ∠ABC lo
cual implica que 4EAD0 ≡ 4ABC, de donde se deduce que el 4D0 EB es isósceles
y ∠BED0 = ∠BEA − ∠D0 EA = 40. Se sigue que ∠EBD0 = 70 y como ∠D0 BA =
∠EBD0 − ∠ABE = 10, resulta que D0 = D y por lo tanto BC = AD0 = AD.
Figura 106
78
Cuadriláteros.
1. Sea ABCD un paralelogramo. Se construyen triángulos equiláteros exteriores 4CDP y
4ADQ, como se muestra en la figura 107. Demuestre que el 4BP Q es equilátero.
Figura 107
Solución: Observe que al hacer una rotación de centro P y ángulo 60, el triángulo
P BC se transforma en el triángulo P QD (observe los segmentos P C y CB tras esta
transformación), mientras que al hacer una rotación de centro Q y ángulo 60, el triángulo
P QD se transforma en triángulo BQA. Como la rotación mantiene las distancias, P B =
P Q = BQ, por lo que el trı́angulo BP Q es equilátero.33
2. (II Olimpiada Matemática del Cono Sur) En la figura 108 ABCD y AECF son paralelogramos. Demuestre que BEDF es paralelogramo.
Figura 108
33
Una demostración más rigurosa se basa en el cálculo de los ángulos ∠P CB = ∠P DQ = ∠BAQ = 120 +
∠ABC y en la utilización del criterio LAL para justificar 4P CB ≡ 4P DQ ≡ 4BAQ.
79
Solución 1: Sea M el punto medio de AC. Las diagonales AC y BD se bisecan en M ,
mientras que las diagonales AC y EF también se bisecan en M , entonces BD y EF se
bisecan en M por lo que BEDF es un paralelogramo.
Solución 2: Como AD k CB y AE k CF entonces ∠DAE = ∠BCF . Entonces, por
propiedades de paralelogramos ∠BAE = ∠BAD −∠EAD = ∠BCD −∠BCF = ∠F CD;
además, AB = CD y AE = CF . Por criterio LAL, 4BAE ≡ 4DCF , y entonces BE =
DF . Análogamente se demuestre que 4ABF ≡ 4CDE, lo cual implica BF = DE.
Como BEDF es un cuadrilátero con lados opuestos iguales, es un paralelogramo.
3. ABCD es un cuadrilátero convexo y O es un punto en su interior. Sean P , Q, R, S,
los puntos medios de los lados AB, BC, CD, DA, respectivamente. Por P se traza una
paralela a OR, por Q se traza una paralela a OS, por R se traza una paralela a OP , y
por S se traza una paralela a OQ. Demuestre que estas cuatro rectas concurren.
Solución: Al tomar las rectas OP y OR y sus paralelas se forma el paralelogramo P ORM ,
y al tomar las rectas OQ y OS y sus paralelas se forma el paralelogramo OQN S. Por el
teorema de Varignon, sabemos que P QRS es un paralelogramo, y llamaremos T al punto
de corte de sus diagonales. Observe que el punto de corte de las diagonales de P ORM es
el punto medio de P R, i.e., T ; análogamente, el punto de corte de las digonales de OQN S
es el punto medio de SQ, i.e., T nuevamente. Ası́, M es la reflexión de O con respecto a
T , y de igual forma queda definido N , por lo que M = N y las cuatro rectas concurren.
4. (Héctor Alberti) Sea ABCD un cuadrado. Se construyen los triángulos equiláteros BDA0 ,
ACB 0 , BDC 0 y ACD0 (Véase figura 109). Demuestre que el A0 B 0 C 0 D0 es también un
cuadrado.
Figura 109
Solución: Como A0 B = A0 D, AB = AD, CB = CD, C 0 B = C 0 D, los puntos A0 , A,
C, C 0 pertenecen a la mediatriz de BD, y por tanto, están alineados. Análogamente, B 0 ,
B, D, D0 están alineados; por lo tanto A0 C 0 ⊥ B 0 D0 . Por otra parte, si O es el centro
de ABCD, como los triángulos equiláteros construidos son todos iguales (tienen lados
iguales a la diagonal de ABCD) de altura h, OA0 = OB 0 = OC 0 = OD0 = h. Entonces,
A0 B 0 C 0 D0 es un cuadrilátero con diagones que se bisecan en O (es paralelogramo), son
iguales A0 C 0 = B 0 D0 = 2h (es rectángulo) y son perpendiculares (es rombo), lo cual
implica que es cuadrado.
80
5. Un trapecio isósceles tiene diagonales perpendiculares y su área es 2010, determine su
altura.
Solución: Considere la figura 110. Sea ABCD el trapecio del problema (AB = CD),
como es trapecio isósceles, es simétrico con respecto a la mediatriz de las bases, en particular, AC = BD. Sean P y Q los pies de las perpendiculares a AD trazadas desde B y
C, respectivamente. Por LAL, 4ABD ≡ 4DCA lo cual implica ∠CAD = ∠BDA = 45
(debido a que AC ⊥ BD) luego en el triángulo rectángulo 4ACQ, ∠ACQ = 45 por lo
que AQ = CQ y es fácil ver que BC = P Q. Luego
(CQ)(BC + AD)
2
(CQ)(P Q + AP + P Q + QD)
=
2
= (CQ)(AP + P Q)
= CQ2
2010 = (ABCD) =
Luego CQ =
√
2010.
Figura 110
6. (IX Competencia de Clubes Cabri, Segunda Ronda) Sea ABCDEF un hexágono regular
cuyo centro es O. Se construyen los cuadrados F SOP y ORCQ. Demuestre que AP QB
y SEDR son rectángulos. Figura 111.
Solución: Por construcción P F = P O = SF = SO, y por propiedades de hexágono
regular34 AF = AO = EF = EO, entonces P , S A, E, pertenecen a la mediatriz de F O
y por tanto, están alineados sobre una recta perpendicular a F O. Análogamente, Q, R,
B, D están alineados sobre una recta perpendicular a CO; además, es fácil demostrar
que AP = BQ = DR = ES. Observe además que AB k CF k DE, lo cual implica
(AB k CF ) ⊥ (AP k BQ), es decir, AP QB es rectángulo, y análogamente para SEDR.
34
Los triángulos OAB, OBC, OCD, ODE, OEF , OF A son equiláteros.
81
Figura 111
7. Sobre los lados del 4ABC se trazan exteriormente los cuadrados ABP Q, CARS y
BCT U . Luego se trazan los paralelogramos AQA0 R, CSC 0 T y BU B 0 P , como en la figura
112.
a) Sean A00 , B 00 , C 00 los centros de los cuadrados BCT U , CARS, ABP Q, respectivamente. Demuestre que estos centros están sobre los lados del 4A0 B 0 C 0 .
b) Demuestre que AA00 , BB 00 , CC 00 concurren.
Solución:
a) Observe que al hacer una rotación de centro A00 y ángulo igual a 90, el 4A00 U B 0 se
transforma en el 4A00 BA, y a la vez este último se transforma en el 4A00 CC 0 (esto es
porque A00 U 7→ A00 B 7→ A00 C y U B 0 7→ BA 7→ CC 0 ); esto significa que A00 B 0 ⊥ A00 A
y A00 A ⊥ A00 C 0 , por lo que B 0 , A00 , C 0 están alineados, es decir, A00 pertenece a B 0 C 0 .
Análogamente se prueban los otros casos.
b) De lo anterior, observe que AA00 es mediatriz de B 0 C 0 , por lo que AA00 , BB 00 , CC 00
concurren en el circuncentro del 4A0 B 0 C 0 .
8. Se dibujan cuadrados exteriores a los lados de un paralelogramo (Vea figura 113), demuestre que:
a) El cuadrilátero determinado por los centros de esos cuadrados es un cuadrado.
b) Las diagonales de ese cuadrado son concurrentes con las del paralelogramo.
Solución:
82
Figura 112
a) Observe que al hacer una rotación de centro O2 y ángulo igual a 90, el 4O2 BO1 se
transforma en el 4O2 CO3 (observe que los segmentos O2 B y BO1 se transforman
en O2 C y CO3 , respectivamente), por lo que O2 O1 = O2 O3 y O2 O1 ⊥ O2 O3 . Repitiendo este razonamiento, O1 O2 = O2 O3 = O3 O4 = O4 O1 y estos segmentos son
perpendiculares si son consecutivos, por lo que O1 O2 O3 O4 es un cuadrado.
b) Basta demostrar que AC y O1 O3 se bisecan,35 y esto es equivalente a demostrar
que AO1 CO3 es un paralelogramo. Esto es cierto porque AO1 = CO3 y AO1 k CO3
(ambos segmentos son perpendiculares a O1 B)
Figura 113
35
Porque ası́ los puntos de corte de las diagonales de ABCD y O1 O2 O3 O4 coincidirı́an.
83
9. Dado un 4ABC, se construyen exteriormente los triángulos rectángulo isósceles 4ACP
y 4BCQ, con AC y BC como hipotenusas. Si M es el punto medio de AB, demuestre
que el 4M P Q también es un triángulo rectángulo isósceles.
Solución: Construya los cuadrados exteriores ACDE y BCF G, como muestra la figura
114. Observe que P y Q son los puntos medios de AD y BF , respectivamente. Al rotar
el 4BCD con centro C y ángulo de 90, se genera el 4F CA, entonces dichos triángulos
son congruentes y en por tanto BD = AF y BD ⊥ AF . Por otra parte, observe que M P
es base media del 4BAD, por lo que 2M P = BD y M P k BD; análogamente, M Q es
base media del 4ABF , por lo que 2M Q = AF y M Q k AF . Por lo tanto M P = M Q y
M P ⊥ M Q.
Figura 114
84
Ángulos en Circunferencia.
1. Dada la figura 115, demuestre que AB k A0 B 0 .
Figura 115
Solución: Observe que los cuadriláteros ABQP y A0 B 0 QP son cı́clicos, por lo que
∠P AB = ∠P QB 0 = 180°−∠P A0 B 0 , por lo tanto AB k A0 B 0 .
2. Dos circunferencias de centros O1 y O2 son tangentes (interna o externamente) en un
punto P ; por este punto se traza una recta que corta nuevamente a la circunferencias en
A y B, respectivamente. Demuestre que AO1 k BO2 .
Solución: En la figura 116 se ha considerado que las circunferencias son tangentes exteriormente, sin embargo, el otro caso es análogo. Se sabe que O1 , P , O2 están alineados, y
que 4AP O1 y BP O2 son triángulos isósceles (dos de sus lados son radios de una circunferencia), entonces ∠O1 AP = ∠AP O1 = ∠BP O2 = ∠O2 BP , por lo que AO1 k BO2 .
Figura 116
3. Dadas dos circunferencias una fuera de la otra como en la figura 117, demuestre que las
tangentes comunes externas forman segmentos iguales; análogamente, las tangentes comunes internas forman segmentos iguales.
Solución:36 Sea P la intersección de las tangentes comunes externas AA0 y BB 0 . Entonces
36
Suponemos que las circunferencias tienen radios distintos; cuando los radios son iguales, el problema se
justifica por la simetrı́a de la figura.
85
AA0 = P A0 − P A = P B 0 − P B = BB 0 . Análogamente se resuelve el caso de las tangentes
comunes internas.
Figura 117
4. Teorema de Pithot. Demuestre que en todo cuadrilátero inscribible, la suma de lados
opuestos es igual.37
Solución: Considere la figura 118, ABCD es el cuadrilátero inscribible, con P , Q, R, S,
los puntos de tangencia sobre AB, BC, CD, DA, respectivamente. Entonces
AB + CD = AP + P B + CR + RD
= AS + BQ + CQ + DS
= BC + DA
Figura 118: Teorema de Pithot.
5. Teorema de Steiner. En todo cuadrilátero exinscrito a una circunferencia, la diferencia
de las longitudes de lados opuestos es igual.
Solución: El cuadrilátero puede quedar en posiciones como las de la figura ??; en ambos
casos, la demostración es muy similar, y análoga a la de Pithot. Para la figura de la
37
El recı́proco de este teorema y del siguiente son también es ciertos.
86
izquierda se tiene que
AB − CD = (AP − BP ) − (CR − RD)
= (AS − BQ) − (CQ − DS)
= AD − BC
Figura 119: Teorema de Steiner.
6. Teorema de Miquel: Dado un 4ABC, sean X, Y , Z puntos sobre AB, BC, CA,
respectivamente . Demuestre que los circuncı́rculos de 4AXZ, 4BY X, 4CZY tienen
un punto en común M .
Solución: Sea M el otro punto de corte de los circuncı́rculos de 4AXZ y 4BY X. Como
los cuadriláteros AXM Z y BY M X son cı́clicos, se tiene
∠Y M Z = 360 − ∠XM Z − ∠Y M X
= 360 − (180 − ∠A) − (180 − ∠B)
= 180 − ∠C
Entonces, CY M Z es cuadrilátero cı́clico, por lo que M está sobre el circuncı́rculo del
4CZY .
Figura 120: Teorema de Miquel.
87
7. Sea ABC un triángulo, y sean L y N las intersecciones de la bisectriz del ángulo A con
el lado BC y el circuncı́rculo de ABC respectivamente (Ver figura 121). Construimos la
intersección M del circuncı́rculo de ABL con el segmento AC. Prueba que los triángulos
BM N y BM C tienen la misma área.
Solución: Observe que ABN C y ABLM son cuadriláteros cı́clicos, por lo que ∠N CB =
∠N AB = ∠LAM = ∠LBM , por lo que CN k BM . Entonces, las distancias de N y C a
la recta BM son iguales, y por tanto, el área del 4BM N es igual al área del 4BM C.
Figura 121
8. Sea AB el diámetro de una semicircunferencia. Se colocan los puntos M y K sobre la
semicircunferencia y sobre AB, respectivamente.38 Sea P el centro de la circunferencia
que pasa por A, K y M ; sea Q el centro de la circunferencia que pasa por B, K y M .
Demuestre que M P KQ es concı́clico.
Solución: Como AB es diámetro, ∠AM B = 90, entonces ∠M AB + ∠M BA = 90. Ası́
∠M P K + ∠M QK
= 2∠M AK + 2∠M BK
2 = 2 (∠M AB + ∠M BA)
= 180
Por lo tanto, M P KQ es concı́clico.
38
M y K son distintos de A y B.
88
9. Las circunferencias Γ1 y Γ2 se cortan en los puntos A y B. Por el punto A se traza una recta
que corta nuevamente a las circunferencias Γ1 y Γ2 en los puntos C y D, respectivamente.
Por los puntos C y D se trazan tangentes a las circunferencias, las cuales se cortan en el
punto M . Demuestra que M CBD es cı́clico. Figura 122.
Solución: Es suficiente probar que M CBD es un cuadrilátero con un par de ángulos
opuestos suplementarios. Por ángulos seminscritos y suma de ángulos internos de un
triángulo, se tiene
∠CM D + ∠CBD = ∠CM D + ∠CBA + ∠DBA
= ∠CM D + ∠M CA + ∠M DA
= 180
Figura 122
10. El 4ABC cumple que ∠A = 90° y AB = AC. Se toma un punto E del segmento AB, se
construye interiormente un triángulo equilátero AEF . EF corta BC en I, y se construye
exteriormente un triángulo equilátero BIJ. Encuentre ∠EJB.
Solución: Como el ∠BJI = 60◦ = ∠AEI,
el cuadrilátero BEIJ es cı́clico, por lo que el
∠EJB = ∠EIB = ∠AEI − ∠EBI = 15°.
89
11. En la figura 123, se sabe que ∠AO1 B − ∠AO2 B = 70◦ y además la tangente EB forma
el triángulo isósceles ABE, con AB = AE. Encuentre ∠EBC.
Figura 123
Solución: Sea ∠AO2 B = 2θ, entonces ∠ACB = θ y por hipóteis ∠AO1 B = 2θ + 70. Por
ángulo seminscrito, ∠ABE = θ + 35, y como el 4ABE es isósceles, ∠AEB = θ + 35.
Finalmente, por la fórmula del ángulo externo aplicada al 4BCE, ∠EBC = ∠AEB −
∠ECB = 35
12. Dos circunferencias Γ1 y Γ2 se cortan en A y B. Una recta por A corta a Γ1 y Γ2 en C y
D, respectivamente, y la paralela a CD por B corta Γ1 y Γ2 en E y F , respectivamente.
(Ver figura 124). Demuestre que 4CDB ≡ 4EAF .
Solución: Sean G = AE ∩ BC y H = AF ∩ BD. Como AC k BE y ACEB es cı́clico,39 ∠CAG = ∠GEB = ∠ACG = ∠GBE = α; análogamente, ∠DAH = ∠HF B =
∠ADH = ∠F BH = β. Observe que 4GAC y 4GBE son triángulos isósceles y por tanto AE = AG+GE = CG+GB = CB; de forma similar se obtiene AF = DB. Finalmente,
∠EAF = 180 − α − β = ∠CBD, por lo que, por el criterio LAL, 4CBD ≡ 4EAF .
13. La Recta de Simson-Wallace. Sean X, Y y Z los pies de las alturas trazadas desde un
punto P en el circuncı́rculo del 4ABC hacia AB, BC y CA, respectivamente. Demuestre
que X, Y y Z están alineados.40
Solución: Como BP Y X es cı́clico, ∠Y XP = Y BP = θ. Como ABP C es cı́clico,
∠CBP = ∠CAP = θ. Como AXP Z es cı́clico, ∠ZAP = ∠ZXP = θ. Por lo tanto,
dado que ∠Y XP = ZXP , los puntos X, Y y Z están alineados.
39
ACEB es un trapecio isósceles.
El recı́proco también es cierto, si X, Y y Z están alineados, entonces P debe estar sobre el circuncı́rculo
del 4ABC; en cualquier otro caso, el 4XY Z se llama el triángulo pedal con respecto al punto P .
40
90
Figura 124
Figura 125: Recta de Simson-Wallace
14. Sea P un punto exterior al cuadrado ABCD tal que ∠AP C = 90◦ , Q es la intersección
de AB y P C, y R el pie de la perpendicular por Q a CA. Demuestre que P , R y D están
alineados.
Solución: Como ∠AP C + ∠ADC = 180°, el cuadrilátero P ADC es cı́clico, entonces
∠AP D = ∠ACD = 45°. Análogamente, como ∠AP Q + ∠ARQ = 180°, el cuadrilátero
P ARQ es cı́clico, entonces ∠AP R = ∠AQR = 90°−∠QAR = 45°. Por lo tanto, como
∠AP D = ∠AP R, los puntos P , R, D están alineados. Figura 126.
15. (OIM 2002, P-4) En un triángulo escaleno ABC se traza la bisectriz interior BD, con D
sobre AC. Sean E y F puntos sobre la recta BD tales que (AE k CF ) ⊥ BD, y sea M el
punto sobre el lado BC tal que DM ⊥ BC. Demuestre que ∠EM D = ∠DM F . Figura
127.
91
Figura 126
Solución: Como DM ⊥ M C y DF ⊥ F C, DF CM es cı́clico, por lo tanto ∠DM F =
∠DCF = θ, y como AE k F C, entonces ∠EAD = ∠DCF = θ. Sea G la intersección
de AE con BC. Como AG ⊥ BE, BE es altura y bisectriz del 4ABG, por lo que este
triángulo es isósceles y además BE es mediatriz de AG; entonces ∠EGD = ∠EAD = θ.
Y finalmente, podemos ver que DEM G es cı́clico, pues ∠DEG = ∠DM G = 90◦ , ası́ que
∠EM D = ∠EGD = θ. De aquı́, el resultado es inmediato.
Figura 127
16. (OMCC 2003, P-2) Sea S una circunferencia y AB un diámetro de ella. Sea t la recta
tangente a S en B y considere dos puntos C y D en t tales que B este entre C y D. Sean
E y F las intersecciones de S con AC y AD y sean G y H las intersecciones de S con
CF y DE. Demuestre que AH = AG.
Solución: Como AEBF es cı́clico (Ver figura 128), ∠AEF = ABF . Luego, como AB ⊥
CD y BF ⊥ AD, se cumple también ∠ABF = ∠F DB, por lo que ∠AEF = ∠F DC, es
decir, el cuadrilátero CDF E es cı́clico. Utilizando este resultado y el hecho que EGHF
también es cı́clico, se tiene ∠EDC = ∠EF G = ∠EHG, por lo que CD k GH. Esto
92
implica que AB ⊥ GH, y como AB pasa por el centro de la circunferencia, debe ser
mediatriz de GH, por lo tanto AG = AH.
Figura 128
17. (The 59th Romanian Mathematical Olympiad District Round) Considere un cuadrado
ABCD y un punto E sobre el lado AB. La diagonal AC corta al segmento DE en el punto P . La perpendicular por P a DE corta al lado BC en F . Probar que EF = AE + CF .
Solución: Se construye E 0 sobre BC de tal manera que CE 0 = AE (como se muestra en la figura 129) y que C quede entre F y E 0 , ası́ por LAL se tiene que los triángulos
rectángulos 4DAE ≡ 4DCE 0 por lo tanto ∠ADE = ∠CDE 0 luego ∠EDE 0 = 90. Por
otra parte, el cuadrilátero DCF P es cı́clico, por lo que ∠P DF = ∠P CF = 45 entonces
∠F DE 0 = ∠EDE 0 − ∠EDF = 45. Ahora por LAL los triángulos 4DEF ≡ 4DE 0 F ,
por lo que EF = E 0 F = E 0 C + CF = AE + CF .
Figura 129
93
18. Teorema de Arquı́medes: En la figura 130, la región delimitada por tres semicircunferencias mutuamente tangentes, es conocida como cuchilla de zapatero o árbelos. Demostrar
que las circunferencias sombreadas son congruentes.
Figura 130: Teorema de Arquı́medes.
Solución: Sean AB, AC, BC los diámetros de las semicircunferencias que forman el árbelos, de radios r, r1 , r2 y centros O, O1 , O2 , respectivamente. De momento nos concentramos
en el lado izquiero de la figura Referenciasfigura58; sea C1 el centro de la circunferencia de
la izquierda y R1 su radio; D, E y F son los puntos de tangencia de esta circunferencia con
dos semicircunferencias del árbelos y con la recta perpendicular a los diámetros por C; finalmente, G es la proyección de C1 sobre AB. En primer lugar, OO1 = OA−O1 A = r−r1 .
Por otra parte, observe que O, C1 , D están alineados y O1 , C1 , E también están alineados,
entonces OC1 = OD − C1 D = r − R1 y O1 C1 = O1 E + EC1 = r1 + R1 . Además, como
CF C1 G es un rectángulo, GC = F C1 = R1 , entonces O1 G = O1 C − GC = r1 − R1 y
OG = O1 G − O1 O = (r1 − R1 ) − (r − r1 ) = 2r1 − (r + R1 ). Ahora, aplicando el teorema
de Pitágoras a 4GO1 C1 y 4GOC1 se tiene
O1 C12 − O1 G2 − C1 G2 = OC12 − OG2 − C1 G2
(r1 + R1 )2 − (r1 − R1 )2 = (r − R1 )2 − (2r1 − (r + R1 ))2
4r1 R1 = −4rR1 − 4r12 + 4r1 r + 4r1 R1
r1 (r − r1 )
⇒ R1 =
r
Figura 131
Análogamente, si r2 y R2 son los radios del semicı́rculo y del cı́rculo de la derecha, respectivamente, entonces
r2 (r − r2 )
R2 =
r
94
Pero 2r = AB = AC + BC = 2r1 + 2r2 , entonces r2 = r − r1 , y sustituyendo en la
ecuación anterior se tiene
R2 =
(r − r1 ) (r − (r − r1 ))
(r − r1 )r1
=
= R1
r
r
95
Teorema de Thales y Semejanza.
1. (IV OMCC, P-4) Sea ABC un triángulo, D el punto medio de BC, E un punto sobre
el segmento AC tal que BE = 2AD y F el punto de intersección de AD con BE. Si
∠CAD = 60°, encuentre la medida de los ángulos del 4F EA. Figura 132.
Solución: Se traza por D una paralela a BE y sea G el punto por el que esta paralela
= AD;
corta al lado AC. Como DG es base media del 4BCE se tiene que DG = BE
2
entonces 4ADG es isósceles y tiene un águlo de 60, por lo que debe ser equilátero.
Finalmente 4AEF también es equilátero, por tanto sus ángulos son iguales a 60.
Figura 132
2. Sea ABCD es un trapecio con AD k BC. M y N son los puntos medios de CD y BC,
M
= 14 , demuestre que
respectivamente, y P el punto común de las rectas AM y DN . Si PAP
ABCD es paralelogramo.
Solución: Sea Q el punto medio de DN , entonces QM k BC k DA. Como M Q es
= CB
. Por otra parte, como 4P M Q ' 4P AD,
base media del 4CDN , M Q = CN
2
4
MQ
PM
1
AD
= AP = 4 , entonces M Q = 4 . Finalmente, como BC k DA y BC = DA, ABCD
AD
es paralelogramo.
3. En la figura 133, BC = CD = DE = EA = x y ∠AEB = 90°. Demuestre que ∠ABC +
∠ACD + ∠ADE = 90°.
Figura 133
√
Solución 1: Por Pitágoras, AD = 2x. Observe que DA2 = 2x2 = DB · DC, por lo que
4ABD ' 4CAD; entonces ∠ABD = ∠CAD y por tanto ∠ABC + ∠ACD + ∠ADE =
96
∠CAD + ∠ACD + ∠ADE = 2∠ADE = 90.
Solución 2: Considere la siguiente cuadrı́cula (Figura 134). Observe que al hacer una
rotación de centro A y ángulo igual a 90, el segmento AC se transforma en AF , por lo
que el 4ACF es triángulo rectángulo isósceles, y ∠ADE = ∠ACF . Se cumple ∠ABC =
∠CF D, porque se forman con la diagonal de tres cuadrados; análogamente, ∠ACD =
∠AF D, porque se forman con la diagonal de dos cuadrados. Sumando los ángulos internos
del 4ACF se obtiene el resultado buscado.
Figura 134
4. (Asiático Pacı́fica) Sea ABC un triángulo y D el pie de la altura con respecto a A. Sean E
y F puntos en una recta que pasa por D (distintos de D) tales que AE ⊥ CE y AF ⊥ BF .
Sean M y N los puntos medios de BC y EF , respectivamente. Demuestre que AN ⊥ N M .
Solución: En la figura 135, AE ⊥ CE y AD ⊥ DC entonces, ADEC es cı́clico, ası́ que
∠DEA = ∠DCA. Del mismo modo, como AF ⊥ BF y AD ⊥ DB, AF BD es cı́clico
y entonces ∠AF D = ∠ABD. Esto implica que 4ABC ' 4AF E y a partir de esta
semejanza, 4ABM ' 4AF N . Luego, ∠AM B = ∠AN F , por lo que el cuadrilátero
AN DM es cı́clico, y por lo tanto ∠AN M = ∠ADM = 90.
97
Figura 135
Puntos y rectas notables.
1. De acuerdo con los datos de la gráfica 136, calcular el valor de AB.
Figura 136
AC
CD
Solución 1: Por el teorema de la bisectrı́z AB
= DB
, de donde AC = 45 x, luego, aplicando
2
el teorema de pitágoras al 4ABC, se tiene que x2 + 182 = 54 x , que después de resolver
se tiene que x = 24.
Solución 2: Dado que D es un punto de la bisectrı́z del ∠BAC, entonces D equidista
de los lados de dicho ángulo, sea pues H ∈ AC talque DH⊥AC y DH = DB = 8
entonces, aplicando el teorema de pitágoras en el 4CDH se deduce que HC = 6, por lo
que AC = x + 6, y por el teorema de pitágoras en el 4ABC, x2 + 182 = (x + 6)2 , por lo
tanto, x = 24.
2. Sea ABCD un paralelogramo. Q es el punto medio de AD, F el pie de la perpendicular
por B sobre QC. Probar que AF = AB.
Solución: Sea E el punto medio de BC y G la intersección de AE con BF . Como
AE k CQ, se tiene que AG ⊥ BF . Pero también, como AE k CQ, entonces EG k CF
por lo que en el 4BCF , EG es base media. Entonces BG = GF de donde se sigue que
4ABF es isósceles porque BG es altura y mediana.
98
3. Sea ABCD un cuadrilátero tal que AB = CD. Las mediatrices de AC y BD se cortan
en P . Probar que ∠P AC = ∠P CA = ∠P BD = ∠P DB. Figura 137.
Solución: Como P está sobre las mediatrices de AC y BD, P A = P C y P B = P D,
y por hipótesis, AB = CD, entonces por criterio LLL, 4ABP ≡ 4CDP . De aquı́,
∠AP B = ∠CP D, entonces ∠BP D = ∠AP D + ∠AP B = ∠AP D + ∠CDP = ∠AP C;
por lo tanto, 4BP D ' 4CP A, dada la igualdad anterior y el hecho que son triángulos
isósceles. De esta semejanza se obtiene ∠P AC = ∠P CA = ∠P BD = ∠P DB.
Figura 137
4. ABC es un triángulo y P un punto en su interior. Sean A0 , B 0 y C 0 las reflexiones de
P sobre BC, CA y AB, respectivamente. D, E y F son los pies de las perpendiculares
respectivos desde A, B y C hacia B 0 C 0 , C 0 A0 y A0 B 0 . Probar que AD, BE y CF son
concurrentes. Figura 138.
Solución: Por propiedades de reflexión axial AC 0 = AP = AB 0 , por lo que el 4AB 0 C 0 es
isósceles, y entonces AD es mediatriz de B 0 C 0 . Análogamente, BE es mediatriz de C 0 A0 ,
mientras que CF es mediatriz de A0 B 0 . Por lo tanto, las rectas AD, BE, CF concurren
en el circuncentro del 4A0 B 0 C 0 .
5. (Arnoldo Aguilar) En la figura 139, ABGH, BCF G y CDEF son cuadrados. Si I es el
centro de ABGH y J = DH ∩ BG, demuestre que I, J y F están alineados.
Solución: Como G es punto medio de HF , BG es una mediana del 4BF H. Además,
BDF H es un paralelogramo, luego sus diagonales BF y DH se cortan en su punto medio,
digamos K. Se sigue que HK es también una mediana del 4BF H, y en consecuencia el
punto de corte de J = KH ∩ BG es el centroide del 4BF H. Pero I es el punto medio de
BH, ası́ que F I es la tercera mediana del 4BF H, por lo tanto J está sobre el segmento
F I.
99
Figura 138
Figura 139
6. (Arnoldo Aguilar) Sea ABC un triángulo equilátero. M y N son los puntos medios de AB
y BC, respectivamente. Exteriormente al 4ABC se construye un triángulo rectángulo
isósceles 4AP C, con ∠AP C = 90◦ . Si I es la intersección de AN y M P , demuestre que
CI es la bisectriz de ∠ACM .
Solución: Observe que AN es bisectriz del ∠BAC.
Como ∠AP C = ∠BM C = 90, el cuadrilátero
AP CM es cı́clico, por lo que ∠P M C = ∠P AC =
∠P CA = ∠P M A = 45, entonces M P es bisectriz
del ∠AM C. De aquı́ se concluye que I = M P ∩ AN
es el incentro del 4ACM , por lo que CI es bisectriz
del ∠ACM .
7. En la figura 140, el 4ABC es tal que ∠A = 90 y ∠B = 60. ¿Cuál es el radio de la
circunferencia?
√
Solución: Por relaciones de triángulos notables, BC = 2 y CA = 3. Sean P y Q
las proyecciones de O sobre AB y AC respectivamente; por construcción, AP OQ es un
rectángulo, pero como OP = r = OQ, es también cuadrado, por lo que AP = r. Observe
100
Figura 140
que la circunferencia es el excı́rculo del 4ABC, por lo que AP = s y entonces
r = s
√
1+2+ 3
=
√2
3+ 3
=
2
8. Dado el paralelogramo ABCD, sea M el punto medio de AB, y N la intersección de CD
con la bisectriz interna del ∠ABC. Demuestre que M C ⊥ BN si y sólo si AN es bisectriz
del ∠DAB.
Solución:
(⇒) Si suponemos que M C ⊥ BN entonces BN es mediatriz de M C, y como BM k CN
entonces ∠CBN = ∠M BN = ∠CN B = ∠M N B, esto implica que BC k M N , y por
tanto N es punto medio de CD; ası́, AM N D es un rombo y AN es bisectriz del ∠DAM .
(⇐) Si suponemos que AN es bisectriz del ∠DAB, es propiedad conocida que AN ⊥ BN ,
por lo que M es el circuncentro del 4ABN y por la relación entre ángulo central y ángulo
inscrito se tiene ∠AM N = 2∠ABN = ∠ABC, por lo tanto M N k BC y BCN M es un
rombo, de donde se obtiene M C ⊥ BN .
9. En el 4ABC, se sabe que los vértices B, C, el circuncentro O y el ortocentro H del
4ABC están todos sobre una misma circunferencia. Figura 141.
a) Calcule el valor de ∠A.
b) Demuestre que el incentro también pertenece al circuncı́rculo de BCOH.
Solución:
a) Sea ∠BAC = α. Como O es el circuncentro del 4ABC, tenemos que ∠BOC = 2α. Por
otra parte, sabemos que al ser H ortocentro, se cumple que ∠BHC = 180◦ − α. Ahora
101
bien, la condición de que B, C, H y O son concı́clicos implica que ∠BOC = ∠BHC,
de donde 2α = 180◦ − α, y por tanto α = 60◦ .
Figura 141
b) Este problema se basa en el siguiente resultado: si I es el incentro del 4ABC entonces
∠BIC = 90 + ∠A
. Como en este caso ∠A = 60, entonces ∠BIC = 120 = ∠BOC =
2
∠BHC, por lo que B, C, O, H, I, se ubican sobre una misma circunferencia.
10. Sea ABC un triángulo tal que las medianas respectivas a B y C son perpendiculares.
Demuestre que se cumple la relación (Ver figura 142).
5BC 2 = CA2 + AB 2 .
Figura 142
Solución: Sean BB 0 y CC 0 las medianas que son perpendiculares, y sea G el centroide.
Observe que el cuadrilátero BCB 0 C 0 tiene diagonales perpendiculares; por el teorema de
Pitágoras se cumple
BC 2 + B 0 C 02 = C 0 B 2 + B 0 C 2
2
2 2
BC
AB
AC
2
=
+
BC +
2
2
2
2
2
2
5BC = AB + AC
102
11. Sea ABC un triángulo de ortocentro H. Sean P y Q los pies de las perpendiculares desde
H a las bisectrices interior y exterior de A, respectivamente. Si M es el punto medio de
BC, mostrar que P , Q y M están alineados. Figura 143.
Solución: Sean E y F los pies de las alturas trazadas desde a B y C, respectivamente.
Se sabe que AP ⊥ AQ, por lo que AP HQ es un rectángulo. Como ∠AP H = ∠AQH =
∠AEH = ∠AF H = 90, los puntos P , Q, E, F , pertenecen a una circunferencia de
diámetro AH. Además, en esta circunferencia, como AP y AQ son bisectrices (interior
y exterior, respectivamente) del ∠EAF , P y Q son los puntos medios del arcos EF ,
por lo que P Q es la mediatriz de EF . Por otra parte, como ∠BEC = ∠BF C = 90, el
cuadrilátero BCEF es cı́clico, y el circuncentro es M , por lo que M E = M F ; entones M
está en la mediatriz de EF , la cual es P Q.
Figura 143
12. En un triángulo ABC, sea M el punto medio de BC. Si se cumple que AB 6= AC y
además ∠M AC + ∠ABC = 90◦ , hallar ∠BAC. Figura 144.
Solución: Sin pérdida de generalidad, suponga que AB > AC. Sea N la intersección de
AB con la mediatriz de BC. Se forma el 4BCN que es isósceles, entonces ∠CN M =
90 − ∠M CN = 90 − ∠M BN = ∠CAN , lo cual implica que el cuadrilátero ACM N es
cı́clico. Por lo tanto, ∠BAC = ∠BM N = 90.
13. Sea ABC un triángulo y U un punto de su circuncı́rculo tal que AU es bisectriz. Las
mediatrices en AB y AC cortan a AU en X y Y . Sea T la intersección de BX con CY .
Demostrar que AU = T B + T C. Figura 145.
Solución:41 Como X y Y pertenecen a las mediatrices de AB y AC, respectivamente,
y a la bisectriz AU , entonces 4ABX y 4ACY cumplen ser isósceles y semejantes entre
si, porque ∠XBA = ∠XAB = ∠Y AC = ∠Y CA = α. Esto implica ∠T XY = ∠XBA +
41
El caso cuando AB = AC es trivial, porque X, Y y T colapsan en el circuncentro del 4ABC.
103
Figura 144
∠XAB = 2α = ∠Y AC+∠Y CA = ∠T Y X, es decir, el 4T XY es isósceles con T X = T Y .
Por otra parte, como ABU C es cı́clico, ∠U BC = ∠U AC = ∠U AB = ∠U CB = α. De
aquı́ se concluye que 4U BC es isósceles, con U B = U C. Además, ∠XU B = ∠ACB =
∠Y CU y ∠XBU = ∠ABC = Y U C; por criterio ALA, 4U XB ≡ 4CY U , por lo que
BX = Y U . Finalmente, T B + T C = (BX − T X) + (CY + T Y ) = Y U + AY = AU .
Figura 145
14. (The 59th Romanian Mathematical Olympiad Final Round) Sea ABCD un rectángulo de
centro O con AB 6= BC. La perpendicular en O a BD corta a las lı́neas AB y BC en los
puntos E y F , respectivamente. Sean M y N los puntos medios de los segmentos CD y
DA, respectivamente. Probar que las lı́neas rectas F M ⊥ EN .
Solución: Considere la figura 146, sin pérdida de generalidad, se ha supuesto AB < BC.42
Sea L el punto medio de AB, y H es la intersección de EF con AD. Se tiene que
LN k BD, y como BD ⊥ EF entonces LN ⊥ EF ; además, como ABCD es un rectángulo, DA ⊥ AB, por lo tanto, H es el ortocentro del 4ELN , y ası́, LH ⊥ EN . Por otra
parte, las reflexiones de L y H con respecto a O son respectivamente M y F , por lo que
LH k M F , lo cual implica que F M ⊥ EN .
42
El otro caso es completamente análogo.
104
Figura 146
15. Sea ABC un triángulo rectángulo, con A = 90◦ . Sea D un punto en su interior tal que
∠DAC = ∠DCA = ∠DBC = α, y AC = BD. Determine el valor de α. Figura 147.
Solución: Sean P y Q los pies de las perpendiculares trazadas desde D hacia CA y AB,
respectivamente, R es un punto sobre BC tal que DB ⊥ DR, y E es la intersección de CD
con AB. Como el 4ACD es isósceles, P es punto medio de AC, entonces AC = 2P A =
2DQ = BD, por lo que el 4BDQ es un triángulo notable y ∠DBQ = 30. Por otra parte,
por criterio ALA, 4ACE ≡ 4DBR, por lo que CE = BR; como P D k AE, D es punto
medio de CE; ası́, si M es el punto medio de BR (y circuncentro del 4BDR) se cumple
que DC = RM = DM , por lo que el 4CDM es isósceles. Por la relación entre el ángulo
inscrito y el ángulo central ∠DM R = 2∠DBR, por lo tanto ∠DCR = 2α. Sumando los
ángulos internos del 4ABC se tiene ∠A + ∠B + ∠C = 90 + 30 + α + 3α = 180, lo cual
implica α = 15.
Figura 147
16. Sea ABC un triángulo y M un punto tal que ∠M AB = 10, ∠M BA = 20, ∠M AC = 40
105
y ∠M CA = 30. Probar que el 4ABC es isósceles. Figura 148.
Solución: Sea D la reflexión del punto A con respecto a la recta BM . Entonces el 4AM D
es isósceles con ∠AM D = 2 (∠M AB + ∠ABM ) = 60 y por lo tanto es equilátero. También ∠DBA = 2∠M BA = 40 y como ∠BAC = 50, implica que DB ⊥ AC. Sea E la
intersección de BD con CM , se cumple que ∠CED = 90 − ∠ACE = 60 = ∠M AD,
por lo que el cuadrilátero AM ED es cı́clico. De aquı́, ∠DEA = ∠DM A = 60. Como
∠DEC = ∠DEA y ED ⊥ AC, se tiene que ED es bisectriz y altura en el 4AEC, por
lo tanto ED es mediatriz de AC, lo cual implica que BA = BC.
Figura 148
17. Teorema de Poncelet: Demuestre si 4ABC es un triángulo rectángulo con ∠A = 90°,
entonces 2(r + R) = b + c.
Figura 149: Teorema de Poncelet
Solución: Sean O e I el circuncentro y el incentro del 4ABC. Como ∠A = 90°, O es el
punto medio de BC, por lo que a = 2R. Por otra parte, si P y Q son las proyecciones de I
sobre AB y AC, claramente AP IQ es rectángulo, pero como I es incentro IP = r = IQ,
por lo que AP IQ es cuadrado. Se sabe que para un triángulo cualquiera AP = s − a, por
lo tanto
r = s−a
b+c−a
r =
2
2r + a = b + c
2(r + R) = b + c
106
18. En la figura 150, ABCD y P QRS son cuadrados, 4ABP ≡ 4BCQ ≡ 4CDR ≡ 4DAS
y los los radios de las cinco circunferencias son iguales a r. Si a es el lado del cuadrado
ABCD, determine r en función de a.
Figura 150
Solución: Se tiene AB = a y se definen b = AP y c = BP ; observe que por las congruencias BQ = b, por lo que P Q = c−b = 2r. Por otra parte (análogamente a la demostración
del teorema de Poncelet), al calcular el inradio del 4ABP se tiene que 2r = b + c − a,
entonces c − b = b + c − a, lo cual implica que a = 2b. Por lo tanto, el 4ABP es un
triángulo notable de 30, 60, 90, y ası́
c−b
2
√
3
a − 12 a
= 2
2
!
√
3−1
=
a
4
r =
107
19. Recta de Euler. El centroide G, el ortocentro H y el circuncentro O de un triángulo
están alineados, y además GH = 2GO.
Figura 151: Recta de Euler
Solución: Considere la siguiente figura. Sean AHa y BHb alturas, OA0 y OB 0 mediatrices.
Observe que HA ⊥ BC y OA0 ⊥ BC, por lo que HA k OA0 ; análogamente HB k OB 0 ;
también, por el teorema de la base media AB k A0 B 0 y AB = 2A0 B 0 . Esto implica que
4ABH ' A0 B 0 O y la razón de semejanza es 2; en particular AH = 2A0 O. Si definimos
G como la intersección de la mediana AA0 con HO, claramente 4AHG ' 4A0 OG y la
razón de semejanza es la misma que la anterior, por lo que GA = 2GA0 , i.e., G es el
centroide del 4ABC. Esto implica que el ortocentro, el centroide y el circuncentro de un
triángulo están alineados, y por la semejanza GH = 2GO.
108

Documentos relacionados